Download as pdf or txt
Download as pdf or txt
You are on page 1of 69

Auditing Theory AT – Quizzer 1

Advanced Review Solutions

================================================================================

“Fundamentals of Auditing and Assurance Services”

1. The single feature that most clearly distinguishes auditing, attestation, and assurance is
a. Type of service.
b. Training required to perform the service.
c. Scope of services.
d. CPA’s approach to the service.

2. The primary goal of the CPA in performing the attest function is to


a. Detect fraud.
b. Examine individual transactions so that the auditor may certify as to their validity.
c. Determine whether the client's assertions are fairly stated.
d. Assure the consistent application of correct accounting procedures.

3. Philippine Standards on Auditing


a. Relate to the filing requirements and enforcement activities of the SEC.
b. Describe procedures to be applied in specific areas of audit activity to eliminate inconsistencies in
audit practice.
c. Are intended to limit the degree of auditor judgment needed to fulfill the attest function.
d. Interpret standards that provide guidelines or measures of quality for an independent audit.

4. The Philippine Standards on Auditing issued by AASC


a. Apply to independent examination of financial statements of any entity when such examination is
conducted for the purpose of expressing an opinion.
b. Must not apply to other related activities of auditors.
c. Need to be applied on all audit related.
d. Require that in no circumstances would an auditor may judge it necessary to depart from a PSA,
even though such a departure may result to more effective achievement of the objective of an
audit.

5. Which of the following best describes the objective of an assurance engagement?


a. To improve the company’s outcomes
b. To compare the company’s information and polices to those of other entities
c. To enhance the credibility of information in order to improve the likelihood that the information will
meet the needs of an intended user
d. To provide consulting and advisory services.

6. Assurance services differ from consulting services in that they


I. Focus on providing advice
II. Involve monitoring of one party by another
a. I only
b. II only
c. Both I and II
d. Neither I nor II

7. As used in PSA, which of the following statements best describes "assertions"?


a. Assertions are the representations of management as to the reliability of the information system.
b. Assertions are the auditor's findings to be communicated in the audit report.
c. Assertions are the representations of management as to the fairness of the financial statements.
d. Assertions are found only in the footnotes to the financial statements.

8. Which of the following mostly describes the function of AASC?


a. To monitor full compliance by auditors to PSAs.
AT Quizzer 1 “Fundamentals of Auditing and Assurance Services 2

b. To assist the Board of Accountancy in conducting administrative proceedings on erring CPAs in


audit practice.
c. To promulgate auditing standards, practices and procedures that shall be generally accepted by
the accounting profession in the Philippines.
d. To undertake continuing research on both auditing and financial accounting in order to make
them responsive to the needs of the public.

9. Which statement does not accurately describe an assurance engagement?


a. The objective of an assurance engagement is for a professional accountant to evaluate or
measure a subject matter that is the responsibility of another party against identified suitable
criteria, and to express a conclusion that provides the intended user with a level of assurance
about that subject matter
b. Not all engagements performed by professional accountants are assurance engagements
c. A particular engagement, to be an assurance engagement, it depends upon whether it exhibits all
the following elements: a two-party relationship, a subject matter, suitable criteria, and a
conclusion
d. An engagement in form of agreed-upon procedures result in the expression of factual findings

10. The Philippine Framework for Assurance Engagements identifies two types of assurance engagement
a practitioner is permitted to perform: a reasonable assurance engagement and a limited assurance
engagement. Which of the following is the objective of a reasonable assurance engagement?
a. A reduction in assurance engagement risk to a level that is acceptable in the circumstances of the
engagement as a basis for a negative form of expression of the practitioner’s conclusion.
b. A reduction in assurance engagement risk to a very low level in the circumstances of the
engagement as a basis for a disclaimer of the practitioner’s conclusion.
c. A reduction in assurance engagement risk to an acceptably low level that is acceptable in the
circumstances of the engagement as a basis for a positive form of expression of the practitioner’s
conclusion.
d. A reduction in assurance engagement risk to a level that is acceptable in the circumstances of the
engagement as a basis for a qualified form of the practitioner’s conclusion.

11. How many members of AASC are needed to approve the exposed draft as Philippine Standards on
Auditing?
a. Majority of the regular members
b. At least eight
c. At least ten
d. At least twelve

12. It refers to the level of satisfaction as to the reliability of an assertion being made by one party for use
by another party
a. Confidence level
b. Assurance level
c. Reasonableness level
d. Tolerable level

13. The following are the elements of an assurance engagement except:


a. Suitable criteria
b. An appropriate subject matter.
c. A two-party relationship involving a practitioner and intended users.
d. Sufficient appropriate evidence.

14. When performing an assurance service, professional accountants use standards or benchmarks to
evaluate or measure the subject matter of an assurance engagement. This element of an assurance
engagement is called:
a. Criteria.
b. Conclusion.
c. GAAP.
d. Assertion.

15. The subject matter of an assertion may include:


a. Historical or prospective financial information.
b. Internal controls.
c. Compliance with regulation.
d. All of the following.

16. In an assurance engagement, the person or class of persons for whom the professional accountant
prepares the report for a specific use or purpose is the:
a. Intended user.
AT Quizzer 1 “Fundamentals of Auditing and Assurance Services 3

b. Responsible party.
c. Management.
d. Client.

17. Which of the following is expected of AASC to do?


a. AASC should normally expose a proposed interpretation of statements
b. AASC should normally expose its opinion on specific queries from a practicing CPA
c. When it is deemed necessary to expose for a comment on proposed interpretations of
statements, the exposure period is understandably shorter than those of the regular drafts of
standards
d. To make the statements on Philippine Standards on Auditing operative, the final statement shall
be submitted to the Board of Accountancy for approval

18. The suitability of the criteria to which the professional accountant will base his evaluation of the
subject matter partly depends on:
a. b. c. d.
Relevance Yes Yes Yes No
Reliability Yes Yes Yes Yes
Understandability Yes No Yes No
Neutrality NO No Yes Yes

19. Professional judgment


a. Should be exercised in planning an performing an audit of financial statements but need not be
documented
b. Can be used as the justification for the decisions made by the auditor that are not supported by the
facts and circumstances of the engagement
c. Is necessary in the evaluation of management’s judgments in applying the entity’s applicable
financial reporting framework
d. Is not used in making decisions about materiality and audit risk

20. Reasonable assurance means


a. Gathering of all available corroborating evidence for the auditor to conclude that there are no
material misstatements in the financial statements, taken as a whole
b. Gathering of the audit evidence necessary for the auditor to conclude that the financial
statements, taken as a whole, are free from any misstatements
c. Gathering of the audit evidence necessary for the auditor to conclude that the financial
statements are free of material unintentional misstatements
d. Gathering of the audit evidence necessary for the auditor to conclude that there are no material
misstatements in the financial statements, taken as a whole

21. An audit in accordance with PSAs is performed on the premise that management and, where
appropriate, those charged with governance have responsibilities that are fundamental to the conduct
of the audit. Which of the following is not one of those responsibilities?
a. To comply with all relevant PSAs in the preparation and presentation of the entity’s financial
statements
b. To provide the auditor with all information, such as records and documentation, and other matters
that are relevant to the preparation and presentation of the financial statements
c. To provide unrestricted access to those within the entity from whom the auditor determines it
necessary to obtain audit evidence
d. To design, implement, and maintain internal control relevant to the preparation and presentation
of financial statements that are free from material misstatement, whether caused by fraud or error

22. The auditor is required to maintain professional skepticism throughout the audit. Which of the following
statements concerning professional skepticism is false?
a. A belief that management and those charged with governance are honest and have integrity
relieves the auditor of the need to maintain professional skepticism
b. Maintaining professional skepticism throughout the audit reduces the risk of using inappropriate
assumptions in determining the nature, timing, and extent of the audit procedures and evaluating
the results thereof
c. Professional skepticism is necessary to the critical assessment of audit evidence
d. Professional skepticism is an attitude that includes questioning contradictory

23. Every independent audit engagement involves both auditing standards and auditing procedures. The
relationship between the two may be illustrated by how they apply from engagement to engagement.
The best representation of this application is that, from one audit engagement to the next
a. Both auditing standards and auditing procedures are applied uniformly
AT Quizzer 1 “Fundamentals of Auditing and Assurance Services 4

b. Auditing standards are applied uniformly but auditing procedures may vary
c. Auditing standards may vary but auditing procedures are applied uniformly
d. Auditing standards are applied uniformly but auditing procedures are optional

24. Generally accepted accounting principles (GAAP) are distinguished from generally auditing standards
(GAAS) in that
a. GAAP are the principles for presentation of financial statements and underlying transactions,
while GAAS are the standards that the auditors should follow when conducting an audit
b. GAAP are the principles auditors follow when conducting an audit, while GAAS are the standards
for presentation of financial statements and underlying transactions
c. GAAP are promulgated by the SEC, while GAAS are promulgated by the FRSC
d. When GAAP are violated, sufficiently strong GAAS may make up for most GAAP deficiencies

25. The primary purpose of an independent financial statement audit is to


a. Provide a basis for assessing management's performance.
b. Comply with state and federal regulatory requirements.
c. Assure management that the financial statements are unbiased and free from material error.
d. Provide users with an unbiased opinion about the fairness of information reported in the financial
statements.

26. Independent auditing can best be described as a


a. Branch of accounting.
b. Discipline that attests to the results of accounting and other operations and data.
c. Professional activity that measures and communicates financial and business data.
d. Regulatory function that prevents the issuance of improper financial information.

27. An independent audit aids in the communication of economic data because the audit
a. Confirms the accuracy of management's financial representations.
b. Lends credibility to the financial statements.
c. Guarantees that financial data are fairly presented.
d. Assures the readers of financial statements that any fraudulent activity has been corrected.

28. Internal auditing often extends beyond examinations leading to the expression of an opinion on the
fairness of financial presentation and includes audits of efficiency, effectiveness, and
a. Internal control.
b. Evaluation.
c. Accuracy.
d. Compliance.

29. Which of the following statements is not a distinction between independent auditing and internal
auditing?
a. Independent auditors represent third party users external to the auditee entity, whereas internal
auditors report directly to management.
b. Although independent auditors strive for both validity and relevance of evidence, internal
auditors are concerned almost exclusively with validity.
c. Internal auditors are employees of the auditee, whereas independent auditors are independent
contractors.
d. The internal auditor's span of coverage goes beyond financial auditing to encompass operational
and performance auditing.

30. The best description of the scope of internal auditing is that it encompasses
a. Primarily operational auditing.
b. Both financial and operational auditing.
c. Primarily the safeguarding of assets and verifying the existence of such assets.
d. Primarily financial auditing.

31. Which of the following best describes the operational audit?


a. It requires the constant review by internal auditors of the administrative controls as they relate to
operations of the company.
b. It concentrates on implementing financial and accounting control in a newly organized company.
c. It attempts and is designed to verify the fair presentation of a company's results of operations.
d. It concentrates on seeking out aspects of operations in which waste would be reduced by the
introduction of controls.

32. The purpose of a compliance audit for a governmental entity is to determine whether
a. Financial statements comply with GAAP and whether the entity is operating efficiently.
b. Financial statements comply with GAAP and the entity has complied with applicable laws and
regulations.
AT Quizzer 1 “Fundamentals of Auditing and Assurance Services 5

c. The entity has complied with applicable laws and regulations.


d. Financial statements comply with GAAP.

33. Government auditing often extends beyond examinations leading to the expression of opinion on the
fairness of financial presentation and includes audits of efficiency, effectiveness, and
a. Internal control
b. Evaluation
c. Accuracy
d. Compliance

34. In government auditing, the three elements of expanded scope auditing are
a. Goal analysis, audit of operations, audit of systems
b. Financial and compliance, economy and efficiency, program results
c. pre-audit, post audit, internal audit
d. national government audit, local government audit, corporation audit

35. An operational audit is designed to


a. Assess the efficiency and effectiveness of management's operating procedures.
b. Assess the presentation of management's financial statements in accordance with generally
accepted accounting principles.
c. Determine whether management has complied with applicable laws and regulations.
d. Determine whether the audit committee of the board of directors is effectively discharging its
responsibility to oversee management's operations.

36. Assurance engagement risk is the risk


a. That the practitioner expresses an inappropriate conclusion when the subject matter information
is materially misstated
b. Of expressing an inappropriate conclusion when the subject matter information is not materially
misstated
c. Through loss from litigation, adverse publicity, or other events arising in connection with a subject
matter reported on
d. Of expressing an inappropriate conclusion when the subject matter information is either materially
misstated or not materially misstated

37. For the purpose of expressing negative assurance in the review report, the practitioner should obtain
sufficient appropriate evidence primarily through
a. Inquiry and confirmation
b. Analytical procedures and substantive tests of details of transactions and account balances
c. Confirmation and tests of controls
d. Inquiry and analytical procedures

38. Which of the following best describes review services?


a. Review engagements focus on providing assurance on the assertions contained in the financial
statements of a public company.
b. Review engagements focus on providing assurance on the internal controls of a public company.
c. Review engagements focus on providing limited assurance on financial statements of a private
company.
d. Review engagements focus on providing advice in a three-party contract

39. In an engagement to perform agreed-upon procedures, an auditor is engaged to


a. Use accounting expertise as opposed to auditing expertise to collect, classify, and summarize
financial information.
b. Provide a moderate level of assurance that the information is free of material misstatement.
c. Provide a high, but not absolute, level of assurance that the information is free of material
misstatement.
d. Carry out those procedures of an audit nature to which the auditor and the entity and any
appropriate third parties have agreed and to report on factual findings.

40. Which of the following statements concerning compilation engagement is incorrect?


a. In a compilation engagement, the accountant is engaged to use accounting expertise as opposed
to auditing expertise to collect, classify and summarize financial information.
AT Quizzer 1 “Fundamentals of Auditing and Assurance Services 6

b. The procedures employed in a compilation engagement enable the accountant to express a


moderate level of assurance on the compiled financial information.
c. Users of the compiled financial information derive some benefit as a result of the accountant’s
involvement because the service has been performed with due professional skill and care.
d. A compilation engagement ordinarily entails reducing detailed data to a manageable and
understandable for without a requirement to test the assertions underlying that information.

41. In a compilation engagement, the accountant is engaged to use accounting expertise as opposed to
auditing expertise to collect, classify, and summarize financial institution. What type of assurance is
provided by the accountant when he/she performs this engagement?
a. Positive assurance
b. Negative assurance
c. No assurance
d. Limited assurance

42. Independence is not a requirement for which of the following engagements?


Compilation Review Agreed-upon Procedures
a. No Yes No
b. No No No
c. Yes No Yes
d. Yes Yes Yes

43. Which of the following engagements require compliance with the requirements of the Code of Ethics
for Professional Accountants in the Philippines?
Compilation Review Agreed-upon Procedures
a. No Yes No
b. No No No
c. Yes No Ye
d. Yes Yes Yes

44. Which of the following statements is correct concerning an auditor’s responsibilities regarding
financial statements?
a. An auditor’s responsibilities for audited financial statements are confined to the expression of the
auditor’s opinion.
b. The fair presentation of audited financial statements in conformity with GAAP is an implicit part of
the auditor’s responsibilities.
c. Making suggestions that are adopted about the form and content of an entity’s financial
statements impairs an auditor’s independence.
d. The auditor’s report should provide an assurance as to the future viability of the entity.

45. Which of the following best describes why an independent auditor is asked to express an opinion on
the fair presentation of financial statements?
a. It is difficult to prepare financial statements that fairly present a company’s financial position, cash
flow, and operations without the expertise of an independent auditor
b. It is management’s responsibility to seek available independent aid in the appraisal of the
financial information shown in its financial statements
c. The opinion of an independent party is needed because a company may not be objective with
respect to its own financial statements
d. It is customary courtesy that all stockholders of a company receive an independent report on
management’s stewardship in managing the affairs of the business

46. When a CPA expresses an opinion on financial statements, his or her responsibilities extend to
a. The underlying wisdom of the client’s management decisions.
b. Whether the results of the client’s operating decisions are fairly presented in the financial
statements.
c. Active participation in the implementation of the advice given to the client.
d. A ongoing responsibility for the client’s solvency.

47. Reducing assurance engagement risk to zero is very rarely attainable or cost beneficial as a result of
the following factors, except
AT Quizzer 1 “Fundamentals of Auditing and Assurance Services 7

a. The use of selective testing


b. The fact that much of the evidence available to the practitioner is persuasive rather than
conclusive
c. The practitioner may not have the required assurance knowledge and skills to gather and
evaluate evidence
d. The use of judgment in gathering and evaluating evidence and forming conclusions based on that
evidence

48. In “auditing” accounting data, the concern is with


a. Determining whether recorded information properly reflects the economic events that occurred
during the accounting period.
b. Determining if fraud occurred.
c. Determining if taxable income has been calculated correctly.
d. Analyzing the financial information to be sure that it complies with government requirements.

49. Although a CPA does not guarantee his findings, his opinion is nevertheless valuable to various third
parties. The value of the CPA’s opinion lies in the fact that
a. He has the qualifications required by law to be a CPA.
b. He is under the supervision of the Professional Regulatory Board of Accountancy.
c. He has gathered sufficient, competent, evidential matter to support his opinion.
d. He has followed generally accepted auditing standards.

50. Which of the following statements best describes the primary purpose of Philippine Standards on
Auditing?
a. They are guides intended to set forth auditing procedures that are applicable to a variety of
situations
b. They are procedural outlines which are intended to narrow the areas of inconsistency and
divergence of auditor opinion
c. They are authoritative statements, enforced through the Code of Professional Conduct, that are
intended to limit the degree of auditor’s judgment
d. They are interpretations which are intended to clarify the meaning of “generally accepted auditing
standards”

51. The auditor's judgment concerning the overall fairness of the presentation of financial position, results
of operations, and changes in financial position is applied within the framework of
a. Generally accepted accounting principles.
b. Generally accepted auditing standards.
c. Internal control.
d. Information systems control.

52. The exposure period allowed for each exposure draft of PSA to be considered by the organizations
and persons to whom it is sent for comment is generally
a. Four months
b. Three months
c. Two months
d. Six months

53. The expertise that distinguishes auditors from accountants is in the


a. Ability to interpret generally accepted accounting principles.
b. Requirement to possess education beyond the Bachelor’s degree.
c. Accumulation and interpretation of evidence.
d. Ability to interpret PAS.

54. The financial reporting framework adopted by management and, where appropriate, those charged
with governance in the preparation of the financial statements that is in acceptable in view of the
nature of the entity and the objective of the financial statements, or that is required by law or
regulation.
a. Generally accepted auditing standards.
b. Financial reporting standards
c. Applicable financial reporting framework.
d. Philippine Standards in Auditing.
55. Which of the following statements is INCORRECT?
a. Unlike consulting services, assurance services report on the quality of information.
AT Quizzer 1 “Fundamentals of Auditing and Assurance Services 8

b. Audit engagements encompass assurance engagements.


c. Not all engagements performed by professional accountants are assurance engagements.
d. Non assurance engagements include agreed-upon procedures, compilation of financial or other
information, and preparation of tax returns where no conclusion is expressed, tax consulting and
advisory engagements.

56. In “auditing” financial accounting data, the primary concern is with:


a. Determining whether recorded information properly reflects the economic events that occurred during
the accounting period.
b. Determining if fraud has occurred.
c. Determining if taxable income has been calculated correctly.
d. Analyzing the financial information to be sure that it complies with government requirements.

57. An audit of financial statements is conducted to determine if the


a. Organization is operating efficiently and effectively.
b. Auditee is following specific procedures or rules set down by some higher authority.
c. Overall financial statements are stated in accordance with an identified financial reporting
framework.
d. Client’s internal control is functioning as intended.

58. In determining the primary responsibility of the external auditor for an audit of a company’s financial
statements, the auditor owes primary allegiance to
a. Shareholders, creditors and the investing public.
b. The management of the audit client because the auditor is hired and paid by management.
c. The Auditing and Assurance Standards Council, because it determines auditing standards and
auditor’s responsibility.
d. The audit committee of the audit client because the committee is responsible for coordinating and
reviewing all audit activities within the company.

59. An audit involves ascertaining the degree of correspondence between assertions and established
criteria. In the case of an audit of financial statements, which of the following would not be a valid
criterion?
a. International Accounting Standards
b. Philippine Financial Reporting Standards (PFRS)
c. Generally accepted auditing standards
d. PFRS for SMEs

60. Most of the independent auditor’s work in formulating an opinion on financial statements consists of
a. Studying and evaluating internal control
b. Obtaining and examining evidential matter
c. Examining cash transactions
d. Comparing recorded accountability with assets

61. Which of the following is more difficult to evaluate objectively?


a. Efficiency and effectiveness of operations
b. Compliance with applicable government regulations
c. Presentation of financial statements in accordance with the applicable financial reporting criteria.
d. All the given criteria are equally difficult to evaluate effectively.

62. Which of the following best describes the operational audit?


a. It attempts of verifying the fair presentation of a company’s results of operations.
b. It concentrates on implementing financial and accounting control in a newly organized company.
c. It concentrates on seeking out aspects of operations in which waste would be reduced by the
introduction of controls.
d. It requires a constant review of the administrative controls by internal auditors as they relate to
operations of the company.

63. The auditor communicates the results of his or her work through the medium of the
a. Engagement letter
b. Management letter
c. Audit report
d. Financial statements

64. A limited assurance engagement is one in which:


AT Quizzer 1 “Fundamentals of Auditing and Assurance Services 9

a. The aim is a reduction in an assurance engagement risk to n acceptably low level in the
circumstances of the engagement as the basis for a positive form of expression of the
practitioner’s conclusion.
b. The aim is a reduction in assurance engagement risk to a level that is acceptable in the
circumstances of the engagement, but where that risk is greater than for a reasonable assurance
engagement, as the basis for a negative form of expression of the practitioner’s conclusion,
c. The auditor is responsible not only for the conclusion but also for the subject matter.
d. None of the choices apply.

65. Which of the following statements exemplifies positive assurance?


a. “In our opinion internal control is effective, in all material aspects, based on XYZ criteria.”
b. “Based on our work described in this report, nothing has come to our attention that causes us to
believe that internal control is not effective, in all material respects, based on XYZ criteria.”
c. Both a and b.
d. Neither a nor b.

66. The three commonly-sought types of assurance services are:


a. Audits, review and compilations.
b. Audits, compilations and other assurance services.
c. Reviews, compilations and other assurance services.
d. Audits, reviews and other assurance services.

67. This assurance engagement provides a limited level of assurance about the client’s historical financial
statements:
a. Financial statements audit
b. Review of historical financial statements
c. Forecasts and projections
d. Agreed-upon procedures

68. Which of the following best describes risk assessment services?


a. An engagement which provides assurance about whether financial and non-financial information
being reported from the entity’s performance measurement system is reliable.
b. An engagement which involves the evaluation of the quality of health care, medical services and
looks into the health care delivery system.
c. An engagement which identifies a set of risks that affect the client organization. It involves the
study of the link between risks and the organization’s vision, mission, objectives and strategies
and the development of new and relevant measures.
d. None of the following.

69. Which of the following best describes business performance measurement?


a. An engagement which identifies a set of risks that affect the client organization. It involves the
study of the link between risks and the organization’s vision, mission, objectives and strategies
and the development of new and relevant measures.
b. An engagement which provides assurance about whether financial and non-financial information
being reported from the entity’s performance measurement system is reliable.
c. An engagement which involves the evaluation of the quality of health care, medical services and
looks into the health care delivery system.
d. None of the following,

70. In government auditing, the three elements of expanded scope auditing are:
a. Goal analysis, audit of operations, audit of systems
b. Financial and compliance, economy and efficiency, program results
c. Pre-audit, post audit, internal audit
d. National government audit, local government audit, corporation audit

71. An audit designed to determine the extent to which the desired results of an activity established by
the legislative or other authorizing body are being achieved.
a. Economy audit
b. Efficiency audit
c. Program results audit
d. Financial-related audit
72. Which of the following types of audits are most similar?
AT Quizzer 1 “Fundamentals of Auditing and Assurance Services 10

a. Operational audits and compliance audits


b. Independent financial statement audits and operational audits
c. Compliance audits and independent financial statement audits
d. Internal audits and independent financial statement audits

73. Which of the following best describes the reason why independent auditors report on financial
statements?
a. A management fraud may exist and it is more likely to be detected by independent auditors
b. A poorly designed internal control structure may be in existence
c. A misstatement of account balances may exist and is generally corrected as the result of the
independent auditor’s work
d. Different interest may exist between the company preparing the statements and the persons
using the statements.

74. The best statement of the responsibility of the auditor with respect to audited financial statements is:
a. The auditor’s responsibility on fair presentation of financial statements is limited only up to the
date of the audit report.
b. The auditor’s responsibility is confined to his expression of opinion about the audited financial
statements.

c. The responsibility over the financial statements rests with the management and the auditor
assumes responsibility with respect to the notes of financial statements.
d. The auditor is responsible only to his qualified opinion but not for any other type of opinion.

75. The general principles for conducting financial statement audits include which of the following?
a. Compliance with the Code of Ethics for CPAs.
b. Compliance with Philippine Standards on Auditing.
c. Planning and performing the audit with professional skepticism.
d. All of the choices apply.

76. Philippine Standards on Auditing (PSAs) should be looked upon by practitioners as:
a. Ideals to strive for, but which are not achievable.
b. Maximum standards which denote excellent work.
c. Benchmark to be used in all audits, reviews and compilations.
d. Minimum standards of performance which must be achieved on each audit engagement.

77. An auditor needs not abide by a Philippines Standard on Auditing if the auditor believes that
a. The amount is insignificant
b. The requirement of the PSA is impractical to perform
c. The requirement of the PSA is impossible to perform
d. Any of the given three choices is correct

78. Ultimately, the decision about whether or not an auditor is independent must be made by the
a. Auditor
b. Audit committee
c. Client
d. Public

79. Which of the following attributes is required of an auditor in relation to audit clients?
a. Rationalization
b. Bias
c. Loyalty
d. Independence

80. A CPA, while performing an audit, strives to achieve independence in appearance in order to
a. Reduce risk and liability
b. Comply with the generally accepted standards of field work
c. Become independent in fact
d. Maintain public confidence in the profession

81. Which of the following best describes why publicly-traded corporations follow the practice of having
the outside auditor appointed by the board of directors or elected by the stockholders?
a. To comply with the regulations of the Financial Accounting Standards Board
b. To emphasize auditor independence from the management of the corporation
c. To encourage a policy of rotation of the independent auditor
d. To provide the corporate owners with an opportunity to voice their opinion concerning the
quality of the auditing firm selected by the directors
AT Quizzer 1 “Fundamentals of Auditing and Assurance Services 11

82. Practitioner’s independence


a. Minimize risk
b. Helps achieve public confidence
c. Defends against professional liability
d. Achieves compliance with the standards of planning and assessment of risks.

83. An audit committee must be comprised of independent director. Which of the following is
considered an independent director?
a. A member of company management.
b. A retired executive from another company.
c. The company’s independent auditor.
d. A consultant to the company.

84. Theoretically, it is possible to provide an infinite range of assurance from a very low level of
assurance to an absolute level of assurance. In practice, the professional accountants cannot
provide absolute assurance because of the following except,
a.The internal control has its inherent limitations.
b.The professional accountants employ testing process.
c. The lack of expertise of the professional accountants in doing a systematic engagement process.
d.The use of judgment in gathering evidence and drawing conclusions based on that evidence.

85. Which of the following is not one of the limitations of an audit?


a. The use of testing
b. Limitations imposed by client
c. Human error
d. Nature of evidence that the auditor obtains

86. Which of the following statements does not properly describe a limitation of an audit?
a. Many audit conclusions are made on the basis of examining a sample of evidence.
b. Some evidence supporting peso representation in the financial statements must be obtained by
oral or written representation of management.
c. Fatigue can cause auditors to overlook pertinent evidence.
d. Many financial statement assertions cannot be audited.

87. Which of the following is one of the limitations of an audit?


a. The possibility that management may prevent the auditor from performing the necessary audit
procedures.
b. The likelihood that the auditor may not be able to detect material misstatements in the financial
statements because the auditor is engaged only after the year end.
c. The fact that most audit evidence is persuasive rather than conclusive in nature.
d. The risk that the auditor may not possess the training and proficiency required by the
engagement.

88. Which of the following is not a component of an engagement risk?


a. Control risk
b. Inherent risk
c. Business risk
d. Detection risk

89. Auditing is based on the assumption that financial data and statements are
a. In conformity with applicable reporting framework.
b. Verifiable
c. Presented fairly
d. Consistently applied

90. The decision of whether the criteria are suitable involves considering whether the subject matter of
the assurance engagement is capable of reasonably consistent evaluation or measurement using
such criteria. Which of the following characteristics is not considered necessary in determining
whether the criteria are suitable?
a. Relevance
b. Neutrality
c. Reliability
d. Sufficiency
91. Choose one of the following which would describe best the phrase “Philippine Standards on Auditing
(PSAs)”.
AT Quizzer 1 “Fundamentals of Auditing and Assurance Services 12

a. They identify the policies and procedures for the conduct of the audit
b. They define the nature and extent of the auditor’s responsibilities
c. They provide guidance to the auditor with respect to planning the audit and writing the audit
report
d. They set forth a measure of the quality of the performance of audit procedures

92. The PSAs established by the Board of Accountancy apply


a. Only to the PICPA membership
b. To all CPAs
c. Only to those who choose to follow them
d. Only when conducting audits subject to the Board’s jurisdiction

93. On every audit engagement, the CPA should comply with applicable PSA
a. Without exception.
b. Except in examinations that result in a qualified report.
c. Except in engagements where the CPA is associated with unaudited financial statements.
d. Except in examinations of interim financial statements.

94. One of the requirements of the Philippine Standards on Auditing is “sufficient and appropriate
evidential matter should be obtained.” The term “appropriate” refers primarily to
a. Relevance of the evidence.
b. Measurability of the evidence.
c. Consistency of the evidence.
d. Dependability of the evidence.

95. The sufficiency and appropriateness of evidential matter is determined by


a. Philippine Standards on Auditing
b. Judgment of the auditor
c. Availability of corroborating data
d. Relevance of evidential matter

96. In general, internal auditor’s independence will be greatest when they report directly to the:
a. financial vice-president
b. corporate controller
c. audit committee of the board of directors
d. corporate stockholders

97. Operational audits often have an objective of determining whether an entity’s


a. Internal control is adequately operating as designed.
b. Operational information is in accordance with generally accepted governmental auditing
standards.
c. Financial statements present fairly the results of operations.
d. Specific operating units are functioning efficiently and effectively.

98. An audit designed to detect violation of laws and regulations would be referred to as
a. A financial statement audit
b. A compliance audit
c. A performance audit
d. An operational audit

99. Which of the following attributes is more closely associated with attestation services performed by a
CPA firm than with other lines of professional work?
a. Integrity
b. Competence
c. Independence
d. Keeping informed on current professional developments

100. Professional skepticism dictates that when management makes a statement to the auditors, the
auditors should
a. Disregard the statement because it ranks low of the evidence quality scale.
b. Corroborate the evidence with other supporting documentation whenever possible.
c. Require that the statement be put in writing.
d. Believe the statement in order to maintain the professional client-auditor relationship.

***end of AT1**
Auditing Theory AT – Quizzer 2
Advanced Review Solutions
=========================================================================

“PROFESSIONAL PRACTICE OF ACCOUNTING”

1. The primary duty to enforce the provisions of RA 9298 and its IRR rests with
a. The PRC.
b. The PRC and the BOA.
c. The BOA.
d. The AASC.

2. Which of the following is not an objective of the Philippine Accountancy Act of 2004?
a. The standardization and regulation of accounting education
b. The examination for registration of certified public accountants
c. The supervision, control and regulation of the practice of the accountancy in the
Philippines.
d. The development and improvement of accounting standards that will be generally
accepted in the Philippines.

3. The practice of accountancy includes


a. Practice of Public Accountancy.
b. Practice in Commerce and Industry.
c. Practice in Education/Academe.
d. All of the above

4. Which of the following does not constitute a practice of accountancy?


a. A person holding out himself as one skilled in the knowledge, science and
practice of accounting and as qualified person to render professional services as
a CPA to more than one client.
b. A person representing his/her employer before government agencies on tax and
other accounting related matters.
c. A person in educational institution teaching accounting, auditing, business law,
taxation or other technically related subjects.
d. A person is appointed as a marketing director of a government-owned and
controlled corporation.

5. It is an organization engaged in the practice of public accountancy, consisting of a sole


proprietor , either alone or with one or more staff member(s)
a. Sector
b. Partnership
c. Firm
d. Association

6. A CPA is in public practice when he/she


a. Represents his/her employer before government agencies on tax and other
matters related to accounting.
b. Represents his/her clients before government agencies on tax and other matters
related to accounting.
c. Teaches accounting, auditing, management advisory services, accounting aspect
of finance, business law and other technically related subjects.
d. Holds, or is appointed to, in an accounting professional group in government or in
a government-owned and/or controlled corporation where decision making
requires professional knowledge in the science of accounting.

7. Which of the following statements concerning the practice of accountancy in commerce and
industry is incorrect?
a. A CPA is in the practice of accountancy in commerce and industry when he or
she is involved in decision making requiring professional knowledge in the
science of accounting, as well as the accounting aspects of finance and taxation.
b. A CPA is in the practice of accountancy in commerce and industry when he/she
represents his/her employer before government agencies on tax and other
matters related to accounting.
c. A CPA is in the practice of accountancy in commerce and industry when such
employment or position requires that the holder thereof must be a Certified Public
Accountant.
d. A CPA is in the practice of accountancy in commerce and industry when
he/she renders professional services as a Certified Public Accountant to
more than one client on a fee basis.
AT Quizzer 2 – RA9298 and its IRR _ 2

8. This is the integrated national professional organization of Certified Public Accountants


accredited by the Professional Board of Accountancy and Professional Regulations
Commission:
a. Association of CPAs in Public Practice
b. Accredited National Professional Organization of Certified Public Accountants
c. Association of Professional Certified Public Accountants
d. Organization of Certified Public Accountants in the Philippines

9. The members of the Professional Regulatory Board of Accountancy shall be appointed by


the
a. Philippine Institute of CPAs (PICPA).
b. Professional Regulation Commission (PRC).
c. President of the Philippines.
d. Association of CPAs in Public Practice (ACPAPP).

10. Which of the following is not a qualification of a member of the Board of Accountancy?
a. He/she must be of good moral character and must not have been convicted of
crimes involving moral turpitude.
b. He/she must be a duly registered CPA with at least ten (10) years of work
experience in the practice of public accounting.
c. He/she must be a natural-born citizen and a resident of the Philippines.
d. He/she must not be a director or officer of the APO at the time of his/her
appointment.

11. The following statements relate to the term of office of the chairman and members of the
Board of Accountancy (BOA). Which is false?
a. The chairman and members of the BOA shall hold office for a term of three (3)
years.
b. Any vacancy occurring within the term of a member shall be filled up for the
unexpired portion of the term only.
c. No person who has served two successive complete terms as chairman or
member shall be eligible for reappointment until the lapse of two (2) years.
d. Appointment to fill up an unexpired term is not to be considered as a complete
term.

12. The Board of Accountancy has the power to conduct an oversight into the quality of audits
of financial statements through a review of the quality control measures instituted by
auditors in order to ensure compliance with the accounting and auditing standards and
practices. This power of the BOA is called
a. Quality review.
b. Peer review.
c. Appraisal.
d. Quality control.

13. The chairman and members of the Board of Accountancy shall receive compensation and
allowances comparable to that being received by the chairman and members of the
a. Board of Nursing.
b. Board of Judges in the “Tawag Ng Tanghalan” of ABS – CBN 2.
c. City Councils.
d. International Accounting Standards Board.

14. The president of the Philippines, upon the recommendation of the Commission, may
suspend or remove any member of the Board of Accountancy. Which of the following is not
a valid ground for suspension or removal of members of the Board of Accountancy?
a. Neglect of duty or incompetence.
b. A member of the Board manipulated the CPA licensure examination results.
c. A member of the Board has violated RA9298.
d. A member of the Board has been sued of crimes involving moral turpitude.

15. Which of the following does not fall under the admission requirements for CPA
examinations?
a. A Filipino citizen
b. Is of good moral character
c. Has not been convicted of any criminal offense involving moral turpitude.
d. At least twenty one years of age.

16. The following documents shall be submitted by applicants for the CPA licensure
examination, except:
a. Certificate of Live Birth in National Statistics Office (NSO) security paper.
b. Baptismal Certificate.
c. Marriage contract in NSO security paper for married female applicants.
d. Transcript of records with indication therein of date of graduation and Special
Order number unless it is not required.
AT Quizzer 2 – RA9298 and its IRR _ 3

17. The following statements relate to CPA examination ratings. Which statement is incorrect?
a. To pass the examination, candidates should obtain a general weighted average
of 75% and above, with no rating in any subject less than 65%.
b. Candidates who obtain a rating of 75% and above in at least four (4) subjects
shall receive a conditional credit for the subjects passed.
c. Conditioned candidates shall take an examination in the remaining subject within
three (3) years from the preceding examination.
d. Candidates who fail in two (2) complete CPA examinations may be allowed to
take examinations a third time provided he or she will comply with Section 13 of
the Act.

18. The Board shall submit to the Commission the ratings obtained by each candidate within
how many calendar days after the examination unless extended for just cause?
a. Ten
b. Twenty
c. Thirty
d. One hundred eighty

19. All successful candidates in the CPA examination shall be required to take an oath of
profession before the following except:
a. Any member of the Board
b. Any government official authorized by the Commission
c. Any person authorized by law to administer oaths upon presentation of proof of
his or her qualification
d. A member of the secular organization

20. Which of the following statements concerning the issuance of Certificates of Registration
and Professional Identification Cards to successful examinees is correct?
a. The Certificate of Registration issued to successful examinees is renewable
every three (3) years.
b. The Professional Identification Card issued to successful examinees shall remain
in full force and effect until withdrawn, suspended or revoked in accordance with
RA 9298.
c. The BOA shall not register and issue a Certificate of Registration and
Professional Identification Card to any successful examinee of unsound mind.
d. The BOA, may after the expiration of three (3) years from the date of revocation
of a Certificate of Registration, reinstate the validity of a revoked Certificate of
Registration.

21. Which of the following is not represented in the Auditing and Assurance Standards Council
(AASC)?
a. Board of Accountancy
b. Bangko Sentral ng Pilipinas.
c. Bureau of Internal Revenue.
d. Commission on Audit.

22. The Auditing and Assurance Standards Council (AASC) shall have representatives
from the public sector of the PICPA.
a. 6
b. 7
c. 8
d. 9

23. This standard setting body shall have a chairman who had been or presently a senior
accounting practitioner in any of the scope of account ting practice.
a. FRSC.
b. AASC.
c. PICPA.
d. ACPAPP.

24. This standard setting body shall have a chairman who had been or presently a senior
accounting practitioner in public accountancy.
a. FRSC.
b. AASC.
c. PICPA.
d. ACPAPP.

25. The chairman and the members of the FRSC and AASC shall have a term of
a. 3 years.
b. 5 years.
c. 6 years.
d. 1 year.
AT Quizzer 2 – RA9298 and its IRR _ 4

26. The creation of Financial Reporting Standards Council (FRSC) and Auditing and Assurance
Standards Council (AASC) is intended to assist the Professional Regulatory Board of
Accountancy (Board) in carrying out its function to
a. To monitor the conditions affecting the practice of accountancy and adopt such
measures, rules and regulations and best practices as may be deemed proper
for the enhancement and maintenance of high professional, ethical, accounting
and auditing standards.
b. To supervise the registration, licensure and practice of accountancy in the
Philippines.
c. To prescribe and adopt the rules and regulations necessary for carrying out the
provisions of RA9298.
d. To prepare, adopt, issue or amend the syllabi of the subjects for examinations.

27. How will representation in the Auditing and Assurance Standards Council (AASC) be
distributed from the following organizations?
• Board of Accountancy (BOA)
• Securities and Exchange Commission (SEC)
• Bangko Sentral ng Pilipinas (BSP)
• Commission on Audit (COA)
• An association or organization of CPAs in active public practice of
accountancy
• Accredited National Professional Organization of CPAs (APO)
a. Eight members from the APO and one member from the rest.
b. Nine members from the APO and one member from the rest.
c. Ten members from the APO and one member from the rest.
d. Eleven members from the APO and one member from the rest.

28. The PRC upon the recommendation of the Board of Accountancy (BOA) shall create a
council to assist the BOA in carrying out its powers and functions and in attaining its
objective of continuously upgrading the accountancy education in the Philippines to make
the Filipino CPA globally competitive. This council is to be known as the
a. CPAs in Education Council.
b. Council of CPAs in Education.
c. Education Technical Council.
d. CPAs in the Academe Council.

29. Which of the following is not a ground for suspension or removal of members of the Board?
a. neglect of duty or incompetence
b. intolerance of any violation of the Act
c. final judgment of crimes, involving moral turpitude
d. rigging of the certified public accountant’s licensure examination results

30. The following statements relate to roster of CPAs. Identify the incorrect statement.
a. A roster showing the names and place of business of all registered CPAs shall
be prepared and updated by the Board.
b. Copies of the roster shall be made available to any party as may be deemed
necessary.
c. The Board, upon approval of the Commission, may delegate the preparation of
this roster to the APO.
d. The publication of the roster in the official gazette or in any major news paper of
public circulation shall be deemed compliance with the requirement of RA9298.

31. The following statements relate to the use of seal by registered CPAs. Which is incorrect?
a. A registered CPA shall obtain and use a seal of a design that will suit his/her
taste.
b. The seal should be of a design prescribed by the Board bearing the CPA’s name,
registration number, and title.
c. The auditor’s reports shall be stamped with the CPA’s seal, indicating therein
his/her current Professional Tax Receipt (PTR) number, date/place of payment
when filed with government authorities or when used professionally.
d. The seal of a CPA shall be circular in form.

(Numbers 32 to 34) The seal of a CPA shall be circular in form with a smaller circle. Answer the
following questions that follow using these choices:

a. CPA registration number of the individual CPA, firm or partnership.


b. The letters “CPA”
c. Name of the individual CPA, firm or partnership as the case maybe.
d. The letters “PRC”

32. What is engraved on the upper portion of the space between the circles?
33. What is engraved on the middle portion (inside the smaller circle)?
34. What is engraved on the lower portion of the space between the circles?
AT Quizzer 2 – RA9298 and its IRR _ 5

35. Which of the following shall be issued to examinees who pass the CPA licensure
examination?
a. Certificate of accreditation.
b. Personal identification card.
c. Certificate of registration and professional identification card.
d. Certificate of full compliance and PRC ID.

36. Which of the following statements concerning ownership of working papers is incorrect?
a. All working papers made by a CPA and his/her staff in the course of an
examination remain the property of such CPA in the course of an examination
remain the property of such CPA in the absence of a written agreement between
the CPA and the client to the contrary.
b. Working papers include schedules and memoranda prepared and submitted by
the client of the CPA.
c. Working papers include reports submitted by a CPA to his/her client.
d. Working papers shall be treated confidential and privileged unless such
documents are required to be produced through subpoena issued by a court,
tribunal, or government regulatory or administrative body.

37. Section 36 (Penal Prevision) of RA 9298 states that any person who shall violate any of the
provisions of the Philippine Accountancy Act of 2004 or any of its implementing rules and
regulations shall, upon conviction, be punished by
a. A fine of not less than P50,000.
b. Imprisonment for a period not exceeding two (2) years.
c. A fine of not less than P50,000 or by imprisonment for a period not exceeding
two (2) years.
d. A fine of not less than P50,000 or by imprisonment for a period not exceeding
two (2) years, or both.

38. A professional identification card issued to Certified Public Accountants bearing the
registration number, date of issuance with an expiry date, due for periodic renewal, duly
signed by the Chairperson of the Commission shall have a validity of
a. One year
b. Two years
c. Three years
d. Four years

39. RA 9298 requires that whenever a CPA signs a document in connection with the practice of
his/her profession, he or she must indicate the

Certificate of Professional Identification Professional Tax Tax


Registration Card Receipt No. Identification No.

a. Yes Yes Yes Yes


b. Yes Yes Yes No
c. No Yes No Yes
d. Yes No No No

40. The death or disability of an individual CPA and/or the dissolution and liquidation of a firm
or partnership of CPAs shall be reported to the BOA not later than days from the date
of such death, dissolution or liquidation.
a. 15
b. 30
c. 60
d. 90

41. (Individual CPAs, Firms or Partnerships of CPAs including partners and staff members
thereof shall register with BOA and the PRC. If the application for registration of AB and
Co., CPAs was approved on August 30, 2021, the registration will expire on
a. September 30, 2023
b. December 31, 2023
c. December 31, 2024
d. August 30, 2024

42. Which of the following statements concerning the use of firm or partnership name is
incorrect?
a. In the case of an individual CPA, he/she shall do business under his/her
registered name with the BOA and the PRC and as printed in his/her CPA
certificate (for example, Juan dela Cruz, CPA).
b. In the case of a firm, it shall do business under its duly registered and authorized
firm appearing in the registration documents issued by the Department of Trade
and Industry (DTI) and other government offices and such firm name shall
include the real name of the sole proprietor as printed in his/her certificate (for
example, Juan dela Cruz and Associates).
AT Quizzer 2 – RA9298 and its IRR _ 6

c. In the case of a registered partnership, it shall do business under its name as


indicated in its current Articles of Partnership and Certificate of Registration
issued by the Securities and Exchange Commission (SEC) (For example,
Santos, Reyes and Diaz, CPAs).
d. A CPA shall practice only under an individual firm, or partnership name in
accordance with Philippine laws and shall not include any fictitious name but may
indicate specialization.

43. Which statement is correct regarding CPD (Continuing Professional Development)


requirements for renewal of professional license?
a. The total CPD credit units required for CPAs shall be sixty (60) units for three (3)
years provided that a minimum of fifteen (15) credit units shall be earned in each
year.
b. A registered professional shall be permanently exempted from CPD
requirements upon reaching the age of 65 years old.
c. A registered professional who is working abroad shall be temporarily exempted
from compliance with CPD requirement during his/her stay abroad, provided that
he/she has been out of the country for at least one (1) year immediately prior to
the date of renewal.
d. Those who failed to renew professional licenses for a period of five (5)
continuous years from initial registration, or from last renewal shall be declared
delinquent.

44. A partner surviving the death or withdrawal of all the other partners in a partnership may
continue to practice under the partnership name for a period of not more than years
after becoming a sole proprietor.
a. 1
b. 2
c. 3
d. 4

45. Which of the following is incorrect regarding the qualifications of members of the Board of
Accountancy?
a. Must be resident of the Philippines.
b. Must be a duly registered Certified Public Accountant with at least 10 (ten) years
of work experience in any scope of practice of accountancy.
c. Must be of good moral character and must not have been convicted of crimes
involving moral turpitude.
d. Must not have any pecuniary interest, directly or indirectly, in any school, college,
university where review classes in preparation for the licensure examination are
being offered or conducted.

46. First statement – The Board shall be composed of a chairman and six (6) members
appointed by the President of the Philippines from a list of five (5) recommendees for each
position and ranked by the Commission from a list of eight (8) nominees for each position.

Second statement – The four (4) sectors in the practice of accountancy shall as much as
possible be equitably represented in the Board.

a. First statement is True ; Second statement is False


b. First statement is False ; Second statement is True
c. Both statements are True
d. Both statements are False

47. A CPA certificate is an evidence of


a. Recognition of independence.
b. Basic competence at the time the certificate is granted.
c. Culmination of the education process.
d. Membership in the accredited professional organization – PICPA.

48. These are documents showing the outline embodying topics and concepts of major
subjects prescribed in specific course of study to serve as the basis for test questions in the
CPA licensure examination:
a. Curriculum
b. Outlines
c. Syllabi
d. Prospectuses

49. The Board shall not register and issue a Certificate of Registration to any successful
candidate if
a. Convicted by a court of competent jurisdiction of a criminal offense involving
moral turpitude
b. Guilty of an immoral and dishonorable conduct
c. Of unsound mind
d. All of the above are grounds for the non registration
AT Quizzer 2 – RA9298 and its IRR _ 7

50. Which of the following statements is correct about the PRC CPE Council?
a. The Council shall be composed of six members and a chairperson.
b. The members of the Board shall choose a chairperson among themselves.
c. The members of the Council shall be appointed by the Commission upon
recommendation of the Board in coordination with APO.
d. The chairperson and members of the Council shall have a term of three (3) years
renewable for another term.

51. One credit hour of Continuing Professional Development (CPD) program, activity or source
shall be equivalent to
a. 10 credit units
b. 5 credit units
c. 3 credit units
d. 1 credit unit

52. As defined in Annex “C” of the IRR, this refers to the inculcation, assimilation and
acquisition of knowledge, skills, proficiency and ethical and moral values, after the initial
registration of a professional that raise and enhance the professional’s technical skills and
competence.
a. Professional Development.
b. Continuing Professional Education.
c. Continuing Professional Development.
d. Professional Growth and Development.

53. Which of the following organization has been recognized by the Commission on October 2,
1975, per Accreditation No. 15, as Accredited Professional Organization?
a. Board of Accountancy (BOA).
b. Philippine Institute of CPAs (PICPA).
c. Association of CPAs in Public Practice (ACPAPP).
d. Association of CPAs in Education (ACPAE).

54. The PICPA shall renew its certificate of accreditation once every
a. Two years.
b. Three years.
c. Four years.
d. Five years.

55. A body created by the Board to conduct an oversight of the quality of audit of financial
statements by reviewing the firms quality control is known as
a. Quality Review Committee (QRC).
b. Educational Training Committee (ETC).
c. Financial Reporting Standards Council (FRSC).
d. Auditing and Assurance Standards.

56. Which of the following is one of the functions of QRC?


a. To promulgate accounting and auditing standards that will be generally accepted
in the Philippines.
b. To prescribe and/or adopt a Code of Ethics for the practice of accountancy.
c. To evaluate periodically the performance of educational institutions offering
accountancy education.
d. To conduct a review on applicants for registration to practice public accountancy
and render a report which shall be attached to the application for registration.

57. The APO shall submit nominations with complete documentation to the Commission not
later than prior to the expiry of the term of the incumbent chairman or member.
a. 30 days
b. 60 days
c. 90 days
d. 120 days

58. According to the Philippine Accountancy Act of 2004 (RA 9298), any person who shall
violate RA 9298 or any of its implementing rules and regulations as promulgated by the
Professional Regulatory Board of Accountancy subject to the approval of the PRC, shall
upon conviction, be punished by:
a. A fine of not less than fifty thousand pesos (P50,000) or by imprisonment for a
period of not exceeding three (3) years or both
b. Not less than one hundred thousand pesos (P100,000), or by imprisonment for a
period of not exceeding two (2) years, or both
c. Not less than fifty thousand (P50,000), or by imprisonment for a period not
exceeding two (2) years, or both
d. Not less than one hundred fifty thousand pesos (P150,000), or by imprisonment
for a period of not exceeding three (3) years, or both.
AT Quizzer 2 – RA9298 and its IRR _ 8

59. Within days after the effectivity of RA 9298, the Board, subject to the approval of the
Commission and in coordination with the APO, shall adopt and promulgate such rules and
regulations to carry out the provisions of RA 9298 and which shall be effective days
following their publication in the Official Gazette or in any major daily newspaper of general
circulation.
a. Sixty; thirty
b. Sixty; fifteen
c. Ninety; thirty
d. Ninety; fifteen

60. First statement – It shall be the primary duty of the Commission and the Board to effectively
enforce the provisions of RA 9298 and its Implementing Rules and Regulations.

Second statement – Any person may bring before the Commission, Board, or the
aforementioned officers of the law, cases of illegal practice or violations of RA 9298 and
its Implementing Rules and Regulations.

a. First statement is True ; Second statement is False


b. First statement is False ; Second statement is True
c. Both statements are True
d. Both statements are False

61. Under foreign reciprocity, special/temporary permit may be issued by the Board subject to
the approval of the Commission and payment of the fees the latter has been prescribed
and charged thereof to the following persons except
a. A foreign certified public accountant called for consultation or for a specific
purpose which, in the judgment of the Board, is essential for the development of
the country
b. A foreign certified public accountant engaged as professor, lecturer or critic in
fields essential to accountancy education in the Philippines and his/her
engagement is confined to teaching only
c. A foreign certified public accountant who is internationally recognized expert or
with specialization in any branch of accountancy and his/her service is essential
for the advancement of accountancy in the Philippines.
d. A foreign certified public accountant who visits the Philippines to study additional
learning in the field of computer system and design.

62. The auditing standard setting body created by the Board of Accountancy is known as:
a. Financial Reporting Standards Council (FRSC).
b. Auditing Standards and Practices Council (ASPC).
c. Accounting Standards Council (ASC).
d. Auditing and Assurance Standards Council (AASC).

63. The Chairman and the members of the Financial Reporting Standards Council (FRSC) and
Auditing and Assurance Standards Council (AASC) shall have a term of year(s):
a. One
b. Two
c. Three
d. Six

64. The sector that is most represented in the AASC is the


a. Commerce and Industry
b. Academe
c. Government
d. Public Practice

65. The Board ensures that all higher educational instruction and offering of accountancy,
including accounting review centers and/or CPE providers offering accounting seminars,
comply with policies, standards and requirements of the course prescribed by:
a. PRC
b. PICPA
c. CHED
d. ACPAPP

66. First statement – An individual CPA is a certified public accountant engaged in the practice
of public accountancy under his/her name, by himself/herself only or with one or more staff
members.
Second statement – The area of practice of accountancy namely public accountancy,
commerce and industry, academe/education and government is called a sector.
a. First statement is True ; Second statement is False
b. First statement is False ; Second statement is True
c. Both statements are True
d. Both statements are False
AT Quizzer 2 – RA9298 and its IRR _ 9

67. The Auditing and Assurance Standards Council (AASC) has 17 regular members coming
from the following:
a. Independent auditing firms – 17; PICPA – 9; ACPAPP 1,; Board of Accountancy
– 1; SEC – 1; COA – 1
b. Independent auditing firms – 5; PICPA – 5; ACPAPP -2; Board of Accountancy –
2; SEC – 2; COA – 1
c. Independent auditing firms – 7; PICPA – 6; ACPAPP -1; Board of Accountancy –
1; SEC – 1; COA – 1
d. Independent auditing firms – 5; PICPA – 5; ACPAPP -2; Board of Accountancy –
2; SEC – 2; COA – 1

68. The following statements were based on provisions of RA9298. Select the incorrect
statement.
a. According to the IRR, all sectors in the practice of accountancy shall as much
as possible be equitably represented in the Board.
b. The Accredited National Professional Organization (APO) shall submit its
nominations for the Board of Accountancy (with complete documentation) to
the Commission not later than sixty (60) days prior to the expiry of the term of
an incumbent chairman or member.
c. The vice chairman of the Board of Accountancy shall serve three years as
member of the Board of Accountancy.
d. No person shall serve in the Board of Accountancy for more than six years.

69. The following statements relate to RA9298. Which statement is correct?


a. The Board of Accountancy has the authority to remove, after due process,
any member of the Professional Regulation Commission for negligence,
incompetence or any other just cause.
b. The functions of the Board of Accountancy include the preparation of the
contents of the CPA licensure examination and rating of examination papers.
c. After two years, subject to certain conditions, the Board of Accountancy must
order the reinstatement of a CPA whose certificate of registration has been
revoked.
d. Insanity is a ground for issuing a certificate of registration to a successful
CPA candidate.

70. The following are the objectives of the Continuing Professional Education (CPE) programs
according to Section 32 of IRR of RA9298, except:
a. To raise and maintain the professional’s capability for delivering professional
services.
b. To provide and ensure the continuous education of registered professional
with the latest trends in the profession brought about by the modernization
and scientific and technological advancements.
c. To make the professional locally competitive.
d. To promote the general welfare of the public.

(Numbers 71 and 72). Annex B par 1(a) of the IRR provides that “Individual CPAs, firms or
partnerships of CPAs, including partners and staff members thereof shall register with the
Board of Accountancy and the Professional Regulation Commission.” Santos and Ignacio,
CPAs applied for registration and their application was approved on July 30, 2021.

71. When will the registration expire?


a. September 30, 2023
b. December 31, 2023
c. July 30, 2024
d. December 31, 2024

72. When should Santos and Ignacio, CPAs file for renewal of registration?
a. On or before July 30, 2023
b. On or before September 30, 2023
c. On or before July 30, 2024
d. On or before September 30, 2024

73. The following foreign CPAs may be issued special/temporary permits by the Professional
Regulatory Board of Accountancy, subject to the approval of the Professional Regulation
Commission and payment of the fees the latter has prescribed and charged thereof:
1. A foreign certified public accountant called for consultation or for a specific
purpose which, in the judgment of the Board , is essential for the
development of the country: Provided , That his or her practice shall be
limited only for the particular work that he/she is being engaged
2. A foreign certified public accountant engaged as professor, lecturer or critic
in fields essential to accountancy education in the Philippines and his/her
engagement is confined to teaching only.
AT Quizzer 2 – RA9298 and its IRR _ 10

3. A foreign certified public accountant who is an internationally recognized


expert or with specialization in any branch of accountancy and his/her
service is essential for the advancement of accountancy in the Philippines.
a. I, II, and III
a. I and II
b. I and III
c. II and III

74. According to IRR, this refers to a study, appraisal or review by the Board or its duly
authorized representatives, of the quality of audit of financial statements through a review
of the quality control measures instituted by the individual CPA, firm or partnership of CPAs
engaged in the practice of public accountancy.
a. CPA review
b. Peer review
c. Professional review
d. Quality review

75. A body created by the Board to conduct an oversight of the quality of audit of financial
statements by reviewing the firm’s quality control is known as:
a. Quality Review Committee (QRC).
b. Educational Training Committee (ETC).
c. Financial Reporting Standards Council (FRSC).
d. Auditing and Assurance Standards Council (AASC).

76. Which of the following is one of the functions of the QRC?


a. To promulgate accounting and auditing standards that will be generally accepted
in the Philippines.
b. To prescribe and/or adopt a Code of Ethics for the practice of accountancy.
c. To evaluate periodically the performance of educational institutions offering
accountancy education.
d. To conduct a review on applicants for registration to practice public accountancy
and render a report which shall be attached to the application for registration.

77. “Meaningful experience” in public practice, as defined in the IRR to RA9298, shall mean:
a. At least two years as audit assistant and at least two years as auditor-in-
charge of audit engagements covering full audit functions of significant
clients.
b. At least one year as audit assistant and not more than two years as auditor-
in-charge of audit engagements covering full audit functions of significant
clients.
c. Not more than one year as audit assistant and at least two years as auditor-
in-charge of audit engagements covering full audit functions of significant
clients.
d. At least one year as audit assistant and at least two years as auditor in-
charge of audit engagements covering full audit functions of significant
clients.

78. A certificate of accreditation shall be issued to CPAs in public practice only upon showing,
in accordance with rules and regulations promulgated by the Board and approved by the
PRC, that such registration has acquired a minimum of years meaningful experience
in any of the areas of public practice including taxation.
a. 2
b. 3
c. 4
d. 5

79. Which of the following is expected of AASC to do?


a. AASC should normally expose a proposed interpretation of statements.
b. AASC should normally expose its opinion on specific queries from a practicing
CPA.
c. When it is deemed necessary to expose for a comment on proposed
interpretations of statements, the exposure period is understandably shorter than
those of the regular drafts of standards.
d. To make the statements on Philippine Standards on Auditing operative, the final
statement shall be submitted to the Board of Accountancy for approval
AT Quizzer 2 – RA9298 and its IRR _ 11

80. How will representation in the Financial Reporting Standards Council (FRSC) be distributed
from the following organizations?
• Board of Accountancy (BOA)
• Securities and Exchange Commission (SEC)
• Bangko Sentral ng Pilipinas (BSP)
• Bureau of Internal Revenue (BIR)
• A major organization composed of preparers and users of financial
statements
• Commission on Audit (COA)
• Accredited National Professional Organization (APO)

a. Six members from the APO and one member from the rest
b. Seven members from the APO and one member from the rest.
c. Eight members from the APO and one member from the rest.
d. Nine members from the APO and one member from the rest.

81. Who is not permitted by law to practice accountancy?


a. A corporation whose stockholders are all CPAs.
b. A partnership of CPAs.
c. A sole practitioner.
d. A partnership of CPAS, with some non CPA staff.

82. Any person who shall violate any of the provisions of the Accountancy Act or any of its
implementing rules and regulations promulgated by the Board of Accountancy subject to
the approval of the PRC, shall upon conviction, be punished by
a. Lethal injection.
b. A fine of more than P50,000.
c. Imprisonment for a period not exceeding two years.
d. A fine of not less than P50,000 or by imprisonment for a period not exceeding
two years or both.

83. Affixing the CPA’s seal and signature on the auditor’s report is an indication of:
a. CPA’s acceptance of responsibility for the financial statements audited.
b. Compliance by the CPA of the requisite accounting and auditing standards and
rules.
c. CPA’s accreditation to practice public accountancy.
d. Fair presentation of financial statements audited.

84. The death or disability of an individual CPA and/or the dissolution or liquidation of a firm or
partnership of CPAs shall be reported to the BOA not later than days from the date of
such death, dissolution or liquidation.
a. 15
b. 30
c. 60
d. 90

85. A partner surviving the death or withdrawal of all the partners in a partnership may continue
to practice under the partnership name for a period of not more than years under
becoming a sole proprietor.
a. 1
b. 2
c. 3
d. 4

86. Which of the following statements concerning ownership of working papers is incorrect?
a. All working papers made by a CPA and his/her staff in the course of examination
remain the property of such CPA in the absence of written agreement between
the CPA and the client to the contrary.
b. Working papers include schedules and memoranda prepared and submitted by
the client to the CPA.
c. Working papers include reports submitted by a CPA to his/her client.
d. Working papers shall be treated confidential and privileged unless such
documents are required to be produced through subpoena issued by any court,
tribunal, or government regulatory or administrative body.
87. Which of the following statements about working papers is correct?
a. Working papers prepared by a CPA including those prepared and submitted by
the client are the personal property of the auditor and the client has no right to
these working papers.
b. Working papers may substitute the client’s accounting records.
c. Working papers are subject to confidentiality rules and they can not be shown to
third parties in any circumstances.
d. Working papers should not serve as a reference source for the clients accounting
records.
AT Quizzer 2 – RA9298 and its IRR _ 12

88. The following statements relate to some of the provisions of RA 9298. Which is correct?
a. Audit working papers are generally the property of the company whose financial
statements were audited.
b. After three (3) years, subject to certain conditions, the Board of Accountancy may
order the reinstatement of a CPA whose certificate of registration has been
revoked.
c. The penal provision (Sec. 36) of RA9298 applies only to the violation of any of
the provisions of RA9298 because its Implementing Rules and Regulations are
unenforceable.
d. It shall be the primary duty of the PRC and the BOA to effectively enforce the
provisions of RA9298.

89. The PICPA shall renew its Certificate of Accreditation once every how many years after the
date of the Resolution granting the petition for re-accreditation and the issuance of the said
certificate upon submission of the requirements?
a. 2 years.
b. 3 years.
c. 4 years.
d. 6 years.

90. A CPA firm may not designate itself as “members of the PICPA” unless
a. All of its partners are members of the Institute.
b. Its entire professional staff is members of the Institute.
c. A majority of its professional staff are members of the Institute.
d. At least one partner is a member of the Institute.

*****************
Auditing Theory AT Quizzer 3
Advanced Review Solutions

=============================================================================

2018 CODE OF ETHICS FOR PROFESSIONAL ACCOUNTANTS IN THE PHILS

Multiple Choice:

1. The 2018 Code of Ethics for Professional Accountants in the Philippines is


a. mandatory for all CPAs
b. mandatory for CPAs in public practice
c. mandatory for CPAs in Government
d. not mandatory

2. Which of the following statements best describes why the profession of CPAs has deemed it
essential to promulgate a code of professional ethics and to establish a mechanism for
enforcing observation of the Code?
a. A distinguishing mark of a profession is its acceptance of responsibility to the public
b. A prerequisite to success is the establishment of an ethical code that primarily defines
the professional’s responsibility to clients and colleagues
c. A requirement of most state laws calls for the profession to establish a code of ethics
d. An essential means of self protection for the profession is the establishment of flexible
ethical standards by the profession

3. IFAC stands for:


a. International Foundation of Accountants
b. Integrated Federation for Assurance
c. International Federation of Accountants
d. International Federation of Auditors

4. Suppose that a national statutory requirement in the Philippine setting is in conflict with a
provision of the IFAC code. In this case:
a. Both are applied simultaneously
b. The IFAC Code requirement prevails
c. The national statutory requirement prevails
d. There will be no ruling for that particular requirement

5. Which of the following is not explicitly referred to in the Code of Ethics as source of technical
standards?
a. Commission on Audit (COA)
b. Auditing and Assurance Standards Council (AASC)
c. Securities and Exchange Commission (SEC)
d. Relevant legislation

6. Immediate family includes:


a. parent
b. sibling
c. non-dependent child
d. spouse

7. Close family includes the following, except:


a. Parent
b. Non-dependent child
c. Sibling
d. Spouse
8. Firm includes the following, except:
a. A sole practicing professional accountant
b. An entity that controls a partnership of professional accountants
c. An entity controlled by a partnership of professional accountants
d. A sole practitioner, partnership or corporation of professional accountants

9. Advertising, as defined in the Code of Ethics, means:


a. The communication to the public of facts about a professional accountant which are not
designed for the deliberate promotion of that professional accountant
b. The approach to a potential client for the purpose of offering professional services
c. The communication to the public of information as to the services or skills provided by
professional accountants in public practice with a view to procuring professional business
d. Any of the given choices

10. The following are modifications made to the IFAC Code to consider Philippine regulatory
requirements and circumstances, except:
a. The period for rotation of the lead engagement partner was changed from five to seven
years
b. Advertising and publicity by individual professional accountants in public practice are
generally permitted in the Philippines
c. Additional examples relating to anniversaries and websites wherein publicity is acceptable,
as provided in BOA Resolution 19, Series of 2000, were included
d. Payment and receipt of commissions may be permitted in the Philippines

11. Which statement is incorrect regarding the Code of Ethics for Professional Accountants in the
Philippines?
a. Professional accountants refer to persons who are registered in the PRC as Certified
Public Accountants (CPA) and who holds a valid certificate issued by the Board of
Accountancy
b. Where a national statutory requirement is in conflict with a provision of the IFAC Code, the
IFAC Code requirement prevails
c. The Code of Ethics for Professional Accountants in the Philippines is mandatory for all
CPAs and is applicable to professional services performed in the Philippines on or after
June 30, 2008
d. Professional accountants should consider the ethical requirements as the basic principle
which they should follow in performing their work

12. The following steps are part of the conceptual framework approach. Set the steps in proper
order.
I. Address threats which are other than clearly insignificant through the application of
safeguards
II. Evaluate the significance of threats to compliance with fundamental
principles III. Identify threats to compliance with the fundamental principles
a. I, II, III
b. II, III, I
c. III, I, II
d. III, II, I

13. If a CPA cannot implement appropriate safeguards, the professional accountant should do the
following, except:
a. Decline the specific professional service involved
b. Discontinue the specific professional service involved
c. Resign from the client or the employing organization, as necessary
d. Issue an adverse opinion on the subject matter of the engagement
14. An inadvertent violation of the Code of Ethics, depending on the nature and significance of the
matter, may not compromise compliance with the fundamental principles provided, once the
violation is discovered,
a. The CPA withdraws from the specific professional service involved
b. A disclaimer of opinion is issued to the client as a result of the violation
c. The violation is corrected promptly and any necessary safeguards are applied
d. The engagement is promptly transferred to another professional accountant who has not
committed the violation
15. The state of mind that permits the provision of an opinion without being affected by influences
that compromise professional judgment.
a. Professional skepticism
b. Integrity
c. Objectivity
d. Independence of mind

16. The avoidance of facts and circumstances that are so significant a reasonable and informed
third party, having knowledge, of all relevant information, would reasonably conclude a CPA’s
integrity, objectivity or professional skepticism had been compromised.
a. Principle of segregation
b. Independence in appearance
c. Functional integrity
d. Preemptive estoppel

17. A CPA should maintain objectivity and free of conflicts of interest when performing:
a. Audits, but not any other professional services
b. All attestation services but not other professional services
c. All attestation and tax services, but not other professional services
d. All professional services

18. Franz, CPA, has an overbearing superior, and the superior is suppressing the recording of
certain transactions of the company. Jana is being asked to act contrary to technical and
professional standards. The first remedy in case of ethical conflict is
a. Follow the established policies of the employing organization to seek a resolution of such
conflict
b. Review the conflict problem with the superior
c. Review the conflict problem with the next higher managerial level, after giving notice to the
immediate superior of the intention to do so
d. Seek counseling and advice on a confidential basis with an independent advisor or the
applicable professional accountancy body or regulatory body

19. Prior to beginning the field work on a new audit engagement in which a CPA does not
possesses expertise in the industry in which the client operates, the CPA should:
a. Reduce audit risk by lowering the preliminary levels of materiality
b. Design special substantive tests to compensate for the lack of industry expertise
c. Engage financial experts familiar with the nature of the industry
d. Obtain a knowledge of matters that relate to the nature of the entity’s business

20. The code of professional ethics state that a CPA should maintain integrity and objectivity. The
term
“objectivity” in the Code refers to a CPA’s ability to:
a. choose independently between alternate accounting principles and auditing standards.
b. distinguish between accounting practices that are acceptable and those that are not.
c. be unyielding in all matters dealing with auditing procedures.
d. maintain an impartial attitude on matters that come under the CPA’s review.
21. The principle of professional competence and due care imposes the following obligations on all
professional accountants, except
a. To maintain professional knowledge and skill at the level required to ensure that clients or
employers receive competent professional service; and
b. To act diligently in accordance with applicable technical and professional standards when
providing professional services.
c. To exercise of unsound judgment in applying professional knowledge and skill in the
performance of professional service.
d. To take reasonable steps to ensure that those working under the professional accountant’s
authority in a professional capacity have appropriate training and supervision and where
appropriate, make clients, employers or other users of the accountant’s professional
services aware of the limitations inherent in the services.
22. Which of the following is incorrect regarding professional competence and due care?
a. Professional competence may be divided into three separate phases.
b. The attainment of professional competence requires initially a high standard of general
education.
c. The maintenance of professional competence requires a continuing awareness of
development in the accountancy profession.
d. Diligence encompasses the responsibility to act in accordance with the requirements of an
assignment, carefully, thoroughly and on a timely basis.

23. Which of the following is incorrect regarding confidentiality?


a. The principle of confidentiality imposes an obligation on all professional accountants to
refrain from disclosing outside the firm or employing organization confidential information
acquired as a result of professional and business relationships without proper and specific
authority or unless there is a legal or professional right or duty to disclose and using
confidential information acquired as a result of professional and business relationships to
their personal advantage or the advantage of third parties.
b. The duty of confidentiality ceases after the end of the relationship between the
professional accountant and the client or employer.
c. Confidentiality should always be observed by a professional accountant unless specific
authority has been given to disclose information or there is a legal or professional duty to
disclose.
d. Confidentiality requires that a professional accountant acquiring information in the course
of performing professional services neither uses nor appear to use that information for
personal advantage or for the advantage of a third party.

24. A professional accountant has a professional duty or right to disclose confidential information
in each of the following, except
a. To comply with technical standards and ethics requirements.
b. To disclose to BIR fraudulent scheme committed by the client on payment of income tax.
c. To comply with the quality review of a member body or professional body.
d. To respond to an inquiry or investigation by a member body or regulatory body.

25. Normally the auditor is not permitted to divulge confidential information obtained from a client.
Which of the following situations would be a violation of this requirement?
a. to respond to the information request of a shareholder
b. to respond to a quality review request of the BOA
c. to initiate a complaint with the board of accountancy
d. to ensure adequate disclosure in accordance with PFRSs

26. A CPA shall not disclose confidential information obtained during an audit engagement in
which one of the following situations?
a. When the security of the state so requires
b. With the consent of the client
c. In defense of himself when sued by the client
d. Under the rule against disclosing confidential information
27. In which of the following situations given below would disclosure by a CPA be in violation of
the Code?
a. Disclosing confidential information in order to properly discharge the CPA’s responsibilities
in accordance with his professional standards
b. Disclosing confidential information in compliance with a subpoena issued by a court
c. Disclosing confidential information to another accountant interested in purchasing the
CPA’s practice
d. Disclosing the confidential information in a review of the CPA’s professional practice by the
Quality Review Committee

28. The principle of professional behavior imposes an obligation on professional accountants to


a. Comply with relevant laws and regulations
b. Avoid any action that may bring discredit to the profession
c. Both a and b
d. Neither a nor b

29. In marketing and promoting themselves and their work, professional accountants shall be
honest and truthful. Professional accountants shall not
a. Bring the profession into disrepute.
b. Make exaggerated claims for the services they are able to offer, the qualifications they
possess, or experience they have gained for; or
c. Make disparaging references or unsubstantiated comparisons to the work of others.
d. All of the above

30. The following statements relate to the provisions of the Code of Ethics that deal with the
professional accountant’s marketing of professional services. Which is false?
a. When a professional accountant in public practice solicits new work through advertising or
other forms of marketing, a self-interest threat to compliance with the principle of
professional behavior may be created
b. The professional accountant should be honest and truthful when marketing professional
services
c. Advertising and publicity are generally unacceptable
d. When marketing professional services, the professional accountant should not make
exaggerated claims for services offered, qualifications possessed or experience gained

31. A professional accountant in public practice is allowed to


a. Refer to, use or cite actual or purported testimonials by third parties
b. Publish services in billboard (e.g. tarpaulin, streamers, etc) advertisements
c. Publish and compare fees with other CPAs or CPA firms or compare those services
with those provided by another firm or CPA practitioner
d. Inform interested parties through any medium that a partnership or salaried
employment of an accountancy nature is being sought

32. Which of the following statements concerning publicity is incorrect?


a. Booklets and other documents bearing the name of a professional accountant and giving
technical information for the assistance of staff or clients may be issued to such person,
other professional accountants or other interested parties
b. Professional accountants who author books or articles on professional qualifications;
give the name of their organization; and give any information as to the services that
the firm provides
c. Appropriate newspapers or magazines may be used to inform the public of the
establishment of a new practice, of changes in the composition of a partnership of
professional accountants in public practice, or of any alteration in the address of a
practice
d. A professional accountant may develop and maintain a website in the Internet in such
suitable length and style which may also include announcements, press releases,
publications and such other necessary and factual information

33. The holding of media-covered events undertaken only to commemorate a professional


accountant’s anniversaries in public practice does not violate the rules on advertising and
solicitation provided that
such undertaking should be done only every _____________ years of celebration
a. 5
b. 10
c. 20
d. 25

34. A professional accountant in public practice may issue to clients or, in response to an
unsolicited request, to a non-client
I. A factual and objectively worded of the services provided
II. A directory setting out names of partners, office addresses and names and addresses of
associated firms and correspondents
a. I only
b. II only
c. Both I and II
d. Neither I nor II

35. Which of the following statements concerning publicity is incorrect?


a. A professional accountant may write a letter or make a direct approach to another
professional accountant when seeking employment or professional business
b. Genuine vacancies for staff may be communicated to the public through any medium in
which comparable staff vacancies normally appear
c. Professional accountants are prevented from providing training services to other
professional bodies, associations or educational institutions which run courses for
their members or the public
d. In publications such as those specifically directed to schools and other places of
education to inform students and graduates of career opportunities in the profession,
services offered to the public may be described in a business-like way

36. May occur as a result of the financial or other interests of a professional accountant or of an
immediate or close family member
a. Self-interest threat
b. Self-review threat
c. Advocacy threat
d. Familiarity threat

37. May occur when a previous judgment needs to be reevaluated by the professional accountant
responsible for that judgment
a. Self-interest threat
b. Self-review threat
c. Advocacy threat
d. Familiarity threat

38. May occur when a professional accountant promotes a position or opinion to the point that
subsequent objectivity may be compromised
a. Self-interest threat
b. Self-review threat
c. Advocacy threat
d. Familiarity threat

39. May occur when because of a close relationship, a professional accountant becomes too
sympathetic to the interests of others
a. Self-interest threat
b. Self-review threat
c. Advocacy threat
d. Familiarity threat

40. May occur when a professional accountant may be deferred from acting objectively by
threats, actual or perceived
a. Self-interest threat
b. Intimidation threat
c. Advocacy threat
d. Familiarity threat
41. For assurance engagements provided to an audit client, the following should be
independent of the client:
a. The members of the assurance team
b. The firm
c. Network firms
d. All of these

42. In which of the following situations would a CPA be in violation of the rules of professional
ethics in determining professional fees?
a. A fee based on appropriate rates per hour or per day for the time of each person engaged
in performing professional services
b. A fee which is lower compared to the fee charged in the prior year for similar services
c. A fee based on appropriate rates per hour, where the appropriate rate is based on the
fundamental premise that the organization and conduct of the CPA and the services
provided to clients are well planned, controlled and managed
d. A fee that is based on 10% of the client’s adjusted net income for the current year

43. The rendering of two or more types of professional services concurrently:


a. Is a violation of the Code of Ethics
b. Would impair integrity, objectivity, or independence, or the good reputation of the profession
c. Does not by itself impair integrity, objectivity, or independence
d. Is inconsistent with the practice of public accountancy

44. Ronald, CPA-lawyer has a law practice. Ronald has recommended one of his clients to Dave,
CPA. Dave has agreed to pay Ronald 12% of the fee for services rendered by Dave to
Ronald’s client. Who, if anyone, is in violation of the Code of Ethics?
a. Ronald and Dave
b. Ronald only
c. Dave only
d. Neither of them

45. Which statement is incorrect regarding custody of client’s assets?


a. Client’s assets should not be held by the CPA if there is reason to believe that the assets
were obtained from, or are to be used for, illegal activities
b. A CPA in public practice should maintain one or more bank accounts for clients’ monies
c. Monies may only be drawn from the client account on the instructions of the client
d. Fees due from a client may be drawn from client’s monies without the need of notifying the
client

46. Which statement is incorrect regarding CPAs in business?


a. CPAs in business owe a duty of loyalty to their employer as well as to the profession,
therefore there may be no time that the two will be in conflict
b. A CPA, particularly one having authority over others, should give due weight for the need
for them to develop and hold their own judgment in accounting matters and should deal
with difference of opinion in a professional way
c. When undertaking significant tasks for which a CPA has not had sufficient specific training
or experience, he or she should not mislead the employer as to the degree of expertise or
experience he or she possesses, and where appropriate, expert advice and assistance
should be sought
d. A CPA is expected to present financial information fully, honestly and professionally and
so that it will be understood in its context

47. Which of the following is not a potential benefit of having the auditor provide both auditing and
consulting services to the client?
a. As a consequence of the knowledge gained during the audit, the auditor can provide the
consulting services more efficiently than an outside consultant
b. Information acquired during the course of the consulting engagement may help in the
planning and performing of the audit
c. Systems designed by the auditor as part of a consulting engagement do not need to be
audited as carefully as systems designed by the an outside consultant since the auditor
has prior knowledge of their reliability
d. Consulting practice increases the auditor’s understanding of the client and the client’s
industry.
This understanding reduces the potential for an audit failure

48. One difference between auditors and other professionals is that most professionals:
a. Need not be concerned about remaining independent
b. Don’t have requirements for continuing education beyond college
c. Don’t have to pass a rigorous examination
d. Aren’t expected to act in the public practice

49. S1 A CPA should maintain confidentiality even in a social environment


S2 A CPA should also maintain confidentiality of information disclosed by a prospective client
or employer
a. True, False
b. False, True
c. True, True
d. False, False

50. The nature and significance of threats may differ depending on whether they arise in relation
to the provision of services to a client. In which of the following types of clients would the
Revised Code provide the strictest set of requirements regarding independence?
a. A financial statement audit client
b. A non-financial statement audit assurance client
c. A non-assurance client
d. A non-client

51. Appropriate safeguards during acceptance may include:


a. Obtaining knowledge and understanding of the client
b. Obtaining knowledge and understanding about the client’s owners, managers and those
responsible for its governance and business activities
c. Securing the client’s commitment to improve corporate governance practices of internal
controls
d. All of these

52. Regarding conflicts of interest, the following safeguards are applicable (select the exception):
a. Notifying the client of the firm’s business interest or activities that may represent a conflict
of interest, and obtaining their consent to act in such circumstances
b. Notifying all known relevant parties that the CPA in public practice is acting for two or more
parties in respect of a matter where their respective interests are in conflict, and obtaining
their consent to so act
c. Notifying the client that the CPA in public practice does not act exclusively for any one
client in the provision of proposed services and obtaining their consent to so act
d. Clear guidelines for members of the client personnel on issues of security and confidentiality

53. A client is seeking a second opinion does not permit the CPA to communicate with the existing
accountant. In such cases, the CPA should:
a. Issue a disclaimer of opinion due to a significant client-imposed scope limitation
b. Consider whether, taking all the circumstances into account, it is appropriate to provide the
opinion sought
c. Consider whether to issue a qualified opinion or disclaimer of opinion due to a significant
client-imposed scope limitation
d. Communicate the client’s refusal directly to the existing accountant

54. The payment or receipt of referral fees or commissions may create threats to which
fundamental principles?
I. Integrity

III. Professional competence and due


care IV. Professional behavior
a. I and III
b. I and IV
c. II, III and IV
d. II and III

55. A CPA in business may be:


a. a salaried employee
b. a partner, director (whether executive or non-executive) or an owner manager
c. a volunteer or another working for one or more employing organization
d. any of these

56. A CPA in business or an immediate or close family member may be offered an inducement
or pressured to offer inducements. Inducements may take various forms, including
a. Gifts and hospitality
b. Preferential treatment
c. Inappropriate appeals to friendship or loyalty
d. Any of these

57. The professional accountant should be constantly conscious of and be alert to factors that give
rise to conflicts of interest. When faced with significant ethical issues such as identifying
unethical behavior or resolving an ethical conflict, the professional accountant should do the
following, except
a. follow the established policies of the employing organization to seek a resolution of such
conflict.
b. review the conflict problem with the immediate superior if the organization’s policies do not
resolve the ethical conflict.
c. seek advice on a confidential basis with an independent advisor or the applicable
professional accountancy body or regulatory body to obtain an understanding of possible
courses of action.
d. report the matter to the Securities and Exchange Commission.

58. When a professional accountant learns of a material error or omission in a tax return of a prior
year, he/she has the responsibility to do the following, except
a. promptly advise the client or employer of the error or omission and recommended that
disclosure be made to the revenue authorities.
b. promptly inform the revenue authorities even without the permission of the client.
c. advise the employer to correct the error and recommend that disclosure be made to the
revenue authorities.
d. consider discontinuing association with the client if the client does not correct the error.

59. This threat to independence occurs when a firm or a member of the assurance firm has a direct
financial interest or material indirect financial interest in an assurance client.
a. Self-Review Threat
b. Advocacy Threat
c. Intimidation Threat
d. Self-Interest Threat

60. This threat to independence occurs when a member of the assurance team has recently
performed services for an assurance client that directly affect the subject matter of the
assurance engagement (e.g., valuation services).
a. Self-Review Threat
b. Self-Interest Threat
c. Advocacy Threat
d. Familiarity Threat

61. This threat to independence occurs when a member of the assurance team has an immediate
family member who is a director or officer of the assurance client.
a. Familiarity Threat
b. Advocacy Threat
c. Self-Review Threat
d. Intimidation Threat

62. Which of the following safeguards to eliminate or reduce threats to independence is provided by
the profession, legislation or regulation?
a. Policies and procedures that emphasize the assurance client’s commitment to fair financial
reporting.
b. Internal policies and procedures to implement compliance with firm policies and procedures
as they relate to independence.
c. Continuing education requirements.
d. Rotation of senior personnel.
63. In which of the following situations would a public accounting firm have violated the Code of
Ethics in determining its fee?
a. A fee is based on whether or not the public accounting firm’s audit report leads to the
approval of the client’s application for bank financing.
b. A fee is to be established at a later date by the Bankruptcy Court.
c. A fee is based upon the nature of the engagement rather than upon the actual time spent on
the engagement.
d. A fee is based on the fee charged by the client’s former auditors.

64. A public accounting firm would least likely be considered in violation of the independence rules
in which of the following instances?
a. A partner’s checking account, which is fully insured by the Philippine Deposit Insurance
Corporation, is held at a financial institution for which the public accounting firm performs
attest services.
b. A manager of the firm donates service as vice president of a charitable organization that is
an audit client of the firm.
c. An attest client owes the firm fees for this and last year’s annual engagements.
d. A covered member’s dependent son owns stock in an attest client.

65. Which of the following is implied when a CPA signs the preparer’s declaration on an income tax
return?
a. The return is not misleading based on all information of which the CPA has knowledge.
b. The return is prepared in accordance with financial reporting standards.
c. The CPA has audited the return.
d. The CPA maintained an impartial mental attitude while preparing the return.

66. The underlying reason for a code of professional conduct for any profession is
a. the need for public confidence in the quality of service of the profession.
b. that it provides a safeguard to keep unscrupulous people out.
c. that it is required by legislation.
d. that it allows licensing agencies to have a yardstick to measure deficient performance.

67. It is not practical for users to evaluate the quality of the performance of most professional
services because of their
a. complexity
b. legal standing
c. veracity
d. salaries and working conditions

68. In a code of conduct, the advantage of general statements of ideal conduct, as opposed to
specific rules of behavior is
a. the emphasis on positive activities.
b. the ability to enforce the ideals.
c. the enforceability of minimum behavior and performance standards.
d. the tendency to define the rules as maximum rather than minimum standards.

69. One difference between auditors and other professionals is that most professionals
a. need not be concerned about remaining independent.
b. don’t have requirements for Continuing Education beyond college.
c. don’t have to pass a rigorous examination.
d. aren’t expected to act in the public interest.

70. “Independence” in auditing means


a. remaining aloof from client.
b. not being financially dependent on client.
c. taking an unbiased viewpoint.
d. being an advocate for the client.

71. Which of the following statements is not true?


a. CPAs lose their independence if they acquire any direct financial interest in a client.
b. CPAs lose their independence if they have a material direct financial interest in a client.
c. CPAs lose their independence if they acquire any indirect financial interest in a client.
d. CPAs lose their independence if they acquire a material indirect financial interest in a client.
72. Although it is possible to take the extreme position that anything affecting either independence
in fact or in appearance must be eliminated to ensure a high level of respect in the community,
the difficulty with this position is that it is likely to restrict significantly
a. the services offered to clients.
b. the freedom of CPAs to practice in the traditional manner.
c. the ability of CPA firms to hire competent staff.
d. all three of the above.

73. An example of an “indirect ownership interest in a client” would be ownership of a client’s stock
by a member’s
a. grandfather
b. spouse
c. dependent child
d. none of the above are “indirect” interests

74. Generally, loans between a CPA firm or its members and an audit client are prohibited because
it is a financial relationship. Which of the following, made under normal lending procedures, is
not an exception to this rule?
a. Immaterial loans
b. Home mortgages
c. Material loans
d. Secured loans

75. Which of the following circumstances would ordinarily not impair the auditor’s independence?
a. Litigation by client against auditor related to tax services.
b. Litigation by client against auditor claiming a deficiency in the previous audit.
c. Litigation by CPA firm against client claiming management fraud or deceit.
d. Intent to start a lawsuit at some future date, after the current audit is completed, claiming a
deficiency in the previous audit.

76. The CPA must not subordinate his or her professional judgment to that of others
a. in every engagement
b. in every audit engagement
c. in every engagement except tax services
d. in every engagement except management advisory services

77. Which of the following would be a violation of the Code of Ethics requiring “objectivity” by the
CPA?
a. The auditor believes that accounts receivable may not be collectible, but accepts
management’s opinion without an independent evaluation.
b. In preparing client’s tax return, the CPA encourages client to take a deduction which the
CPA believes is valid, but for which there is some but not complete support.
c. Both a and b above would be violation.
d. Neither would be violations.

78. Which of the following would be a violation of the Code of Ethics requiring “integrity” by the
CPA?
a. In preparing client’s tax return, the CPA encourages client to take a deduction which the
CPA believes is valid, but for which there is some but not complete support.
b. In preparing client’s tax return, the CPA encourages client to take a deduction for which
there is no support, but which has little chance of discovery by the BIR.
c. Both a and b would be violations.
d. Neither would be violations.

79. The Revised Code of Ethics for Professional Accountants states that a member in public
practice shall not disclose any confidential client information without the specific consent of the
client. This rule would be violated if CPA disclosed information without client’s consent as a
result of
a. a subpoena or summons
b. a peer review
c. a complaint filed with the trial board of the Institute
d. a request by a client’s largest stockholder
80. The confidential relationship will be violated if, without client’s permission, the CPA provides
working papers about client to
a. a court of law which subpoenas them.
b. another CPA firm as part of a PICPA peer review program.
c. another CPA firm which has just purchased the CPA’s entire practice.
d. an investigative or disciplinary body of the PICPA which is conducting a review of the CPA’s
practice.

81. If requested to perform a review engagement for a nonpublic entity in which an accountant has
an immaterial direct financial interest, the accountant is
a. independent because the financial interest is immaterial and, therefore, may issue a review
report.
b. not independent and therefore, may not be associated with the financial statements.
c. not independent, and therefore, may not issue a review report.
d. not independent, and therefore, may issue a review report, but may not issue an auditor’s
opinion.

82. Dave, CPA, has a public accounting practice. He wishes to establish a separate partnership to
offer data processing services to the public and other public accountants.
a. Dave cannot be a partner in any separate partnership which offers data processing services.
b. Dave may form a separate partnership.
c. Dave may form a separate partnership as long as partners are CPAs.
d. Dave may form a separate partnership, but he must give up his public accounting practice.

83. The concept of materiality would be least important to an auditor when considering the
a. decision whether to use positive or negative confirmations of accounts receivable.
b. adequacy of disclosure of a client’s illegal act.
c. discovery of weaknesses in a client’s internal control structure.
d. effects of a direct financial interest in the client upon the CPA’s independence.

84. A self-interest threat may be created, if fees due from an assurance client for professional
services remain unpaid for a long time. Hence, professional fees for prior year’s engagement
must be paid before:
a. the client engages the services of the professional accountant to audit the current year’s
financial statements
b. the professional accountant formulates an opinion on the current year’s financial statements
c. the audit report on the current year’s financial statements are issued
d. the commencement of the current year’s audit engagement

85. Examples of close business relationships that may create self-interest and intimidation threat
least likely include
a. Having a material financial interest in a joint venture with the assurance client or a
controlling owner, director, officer or other individual who performs senior managerial
functions for that client.
b. Arrangements to combine one or more services or products of the firm with one or more
services or products of the assurance client and to market the package with reference to
both parties.
c. Distribution or marketing arrangements under which the firm acts as a distributor or
marketer of the assurance client’s products or services, or the assurance client acts as
the distributor or marketer of the products or services of the firm.
d. The purchase of goods and services from an assurance client by the firm (or from an
audit client by a network firm) or a member of the assurance team, provided the
transaction is in the normal course of business and on an arm’s length basis.

86. Which of the following activities would least likely impair the professional accountant’s
independence?
a. Serving as an officer or director of an audit client
b. Determining which recommendation of the firm should be implemented
c. Being an honorary member of an audit client
d. Reporting, in a management role, to those charged with governance
87. Which of the following would least likely be considered a violation of the independence rules?
a. Receiving a gift from an assurance client
b. Providing a tax consultancy services to an audit client
c. Providing legal services to an assurance client in legal dispute
d. Providing bookkeeping services to an audit client that is listed in the stock exchange

88. Independence of an auditor in relation to the enterprise may be impaired under the following
cases, except
a. Having a direct or material indirect financial interest in the enterprise
b. Connection with the enterprise as a promoter, underwriter, voting trustee, director, officer or
employee
c. Having a loan to or from the enterprise or any officer, director or principal stockholder thereof
with certain exceptions
d. Engaged to render management advisory services to the enterprise

89. An engagement partner who is rotated in the audit of financial statements of listed entity can
only participate on the audit engagement for the same client after a period of
a. Two years
b. Three years
c. Five years
d. Twelve months

90. Which of the following will least likely impair independence?


a. An immediate family member of a member of the assurance team is a director, an officer or an
employee of the assurance client in a position to exert direct and significant influence over the
subject matter of the assurance engagement.
b. A member of the assurance team participates in the assurance engagement while knowing, or
having reason to believe, that he or she is to, may join, the assurance client sometime in the
future.
c. A partner or employee of the firm serves as an officer or as a director on the board of an
assurance client.
d. A partner or an employee of the firm receives a token gift from an assurance client.

91. Contingent fee pricing of public accounting services is


a. Never restricted in public accounting practice.
b. Considered an act discreditable to the profession.
c. Always strictly prohibited in public accounting practice.
d. Prohibited for clients for whom assurance services are provided.

92. In determining estimates of fees, an auditor may take into account each of the following, except
the
a. Value of the service to the client
b. Degree of responsibility assumed by undertaking the engagement
c. Skills required to perform the service
d. Attainment of specific findings

93. The CPA should not undertake an engagement if his fee is to be based upon
a. a percentage of audited net income
b. per diem rates plus expenses
c. the findings of a tax authority
d. the complexity of the service rendered

94. In which of the following situations would a public accounting firm have violated the Code of
Ethics in determining its fee?
a. A fee which is based upon the nature of the engagement rather than upon the actual time
spent on the engagement
b. A fee based on the degree of responsibility that the service entails
c. A fee based on whether or not the audit report leads to the approval of client’s application
for a bank loan
d. A fee that will be established as a result of a bankruptcy proceedings
95. Fees are not to be regarded as contingent if these are:
a. computed as a percentage of the client’s net income
b. computed as a percentage of the amount that the company can save from its tax return
c. dependent on the approval of a bank loan
d. dependent on the resolution of controversy with government agency

96. The provision of legal services by a firm, or network firm, to an entity that is an audit client may
create
Self-review
Advocacy threat threat
a. Yes No
b. No Yes
c. No No
d. Yes Yes

97. A director, an officer or an employee of the assurance client in a position to exert direct and
significant influence over the subject matter of the assurance engagement has been a
member of the assurance
team or partner of the firm. This would situation least likely create
a. self-interest threat
b. self-review threat
c. intimidation threat
d. familiarity threat

98. The approach to a potential client for the purpose of offering professional services is called
a. indecent proposal
b. solicitation
c. encroachment
d. advertising

99. The communication to the public of facts about a professional accountant which are not
designed for the deliberate promotion of that promotional accountant is called
a. advertising
b. publicity
c. solicitation
d. announcement

100. A professional accountant in public practice to whom the existing accounting or the client of
existing accountant has referred audit, accounting, taxation, consulting or similar
appointments, or who is consulted in order to meet the needs of the client is called a(n):
a. management consultant
b. receiving accountant
c. expert
d. successor auditor

**********
Auditing Theory AT – Quizzer 4
Advanced Review Solutions

PSA 200 Revised and Redrafted “Overall Objective of the Independent Auditor and the
Conduct of an Audit in Accordance with PSA”

1. Which of the following has the primary responsibility for the fairness of the representations made in
the financial statements?
a. Client’s management.
b. Audit committee.
c. Independent auditor.
d. Board of Accountancy

2. An audit of financial statements of Toyota Corp. is being conducted by an external auditor. The
external auditor is expected to
a. Express an opinion as to the fairness of Toyota’s financial statements.
b. Express an opinion as to the attractiveness of Toyota for investment purposes.
c. Certify the correctness of Toyota’s financial statements.
d. Examine all evidence supporting Toyota’s financial statements.

3. Which of the following is one of the limitations of an audit?


a. The possibility that management may prevent the auditor from performing the necessary audit
procedures.
b. The likelihood that the auditor may not be able to detect material misstatements in the financial
statements because the auditor is engaged only after the client’s year-end.
c. The fact that most audit evidence is persuasive rather than conclusive in nature.
d. The risk that the auditor may not possess the training and proficiency required by the
engagement.

4. The independent audit is important to readers of financial statements because it


a. Determines the future stewardship of the management of the company whose financial
statements are audited.
b. Measures and communicates financial and business data involved in financial statements.
c. Involves the objective examination of and reporting on management prepared information.
d. Reports on the accuracy of all information in the financial statements.

5. Which of the following is not one of the general principles governing the audit of financial statements?
a. The auditor should plan and perform the audit with an attitude of professional skepticism.
b. The auditor should obtain sufficient appropriate evidence primarily through inquiry and analytical
procedures to be able to draw reasonable conclusion.
c. The auditor should conduct the audit in accordance with PSA.
d. The auditor should comply with the Philippine Code of Professional Ethics.

6. Which one of the following is not among the conditions that give rise to a demand by external users
for independent audits of financial statements?
a. Remoteness of users.
b. Complexity of making economic decisions.
c. Potential conflict of interest between users and preparers of the statements.
d. Consequence for making decisions.

7. The primary reason for audit by an independent, external audit is


a. To satisfy governmental regulatory requirements.
b. To guarantee that there are no misstatements in the financial statements.
c. To provide increased assurance to users as to the fairness of the financial statements.
d. To ensure that any fraud will be discovered.

8. When a CPA expresses an opinion on financial statements, his or her responsibilities extend to
a. The underlying wisdom of the client’s management decisions.
b. Whether the results of the client’s operating decisions are fairly presented in the financial
statements.
c. Active participation in the implementation of the advice given to the client.
d. An ongoing responsibility for the client’s solvency.
AT Quizzer 4 PSA 200 (Rev and Redrafted), 210 Redrafted, 220 Redrafted 2

9. In determining the primary responsibility of the external auditor for a company’s financial statements,
the auditor owes primary allegiance to:
a. The management of the audit client because the auditor is hired and paid by management.
b. The audit committee of the audit client because that committee is responsible for coordinating
and reviewing all audit activities within the company.
c. Stockholders, creditors and the investing public.
d. The Auditing and Assurance Standards Council, because it determines auditing standards and
auditor responsibility.

10. Which of the following would not represent one of the primary problems that would lead to the
demand for independent audits of a company’s financial statements?
a. Management bias in preparing financial statements.
b. The downsizing of business and financial markets.
c. The complexity of transactions affecting financial statements.
d. The remoteness of the user from the organization and thus the inability of the user to directly
obtain financial information from the company.

11. Which of the following is a correct statement?


a. An audit provides limited assurance by attesting to the fairness of the client’s assertions.
b. A review provides positive assurance by attesting the reliability of the client’s assertions.
c. Management consulting services provide attestation in all cases.
d. Accounting services do not provide attestation.

12. Financial statement audits:


a. Reduce the cost of capital.
b. Report on the compliance with laws and regulations.
c. Assess management’s efficiency.
d. Overlook information risk.

13. The audit of historical financial statements should be conducted by the CPA professionals in
accordance with
a. Generally accepted accounting principles.
b. Philippine Standards on Auditing.
c. The auditor’s judgment.
d. The audit program.

14. An audit involves ascertaining the degree of correspondence between assertions and established
criteria. In the case of financial statement audit, which of the following is not a valid criterion?
a. Accounting standards generally accepted in the Philippines.
b. International Accounting Standards.
c. Authoritative financial reporting framework.
d. Philippine Standards on Auditing.

15. The audit process is


a. A special application of the scientific method of inquiry.
b. Regulated by PICPA.
c. The only service a CPA is allowed to perform by law.
d. Performed only by CPAs.

16. In performing a financial statement audit, which of the following would an auditor least likely consider?
a. Internal control.
b. Compliance with GAAP.
c. Quality of management’s business decisions.
d. Fairness of the financial statements amounts.

17. Which of the following is a difference between attestation standards and auditing standards?
a. Attestation standards cover attest engagements, other than those involving GAAP financial
statements.
b. Attestation standards do not require independence in mental attitude.
c. Auditing standards apply only to CPAs while attestation standards apply to all accountants.
d. Attestation standards do not include standards of reporting.

18. Professional skepticism dictates that when management makes statement to the auditors, the
auditors should
a. Disregard the statement because it ranks low of the evidence quality scale.
b. Corroborate the evidence with other supporting documentation whenever possible.
c. Require that the statement be put in writing.
d. Believe the statement in order to maintain the professional client-auditor relationship.
AT Quizzer 4 PSA 200 (Rev and Redrafted), 210 Redrafted, 220 Redrafted 3

19. Information risk refers to the risk that


a. The client’s financial statements may be materially false and misleading.
b. The auditor may express an unqualified opinion on financial statements that are materially
misstated.
c. The client may not be able to remain in business.
d. Errors and frauds would not be detected by the auditor’s procedures.

20. The audit committee of the board of directors of a company is responsible for:
a. Hiring the auditor.
b. Preparing the financial statements.
c. The audit working papers.
d. Independence and obtaining evidence.

21. The single feature that most clearly distinguishes auditing, attestation and assurance is
a. Type of service.
b. Training required to perform the service.
c. Scope of services.
d. CPA’s approach to the service.

22. The Philippine Standards on Auditing (PSAs) require that a report be issued whenever a CPA firm
a. Performs an audit.
b. Is engaged to perform any services of any nature.
c. Is associated with financial statements.
d. Does SEC regulated work.

23. Which one of the following is an example of management expectation for independent auditors?
a. An expert providing a written communication as the product of the engagement.
b. Individuals who perform day-to-day accounting functions on behalf of the company.
c. An active participant in management decision making.
d. An internal source of expertise on financial and other matters.

24. An expectation of the public is that the auditor will recognize that the primary users of audit services
are:
a. The employees.
b. The Securities and Exchange Commission.
c. The investors and creditors.
d. The board of directors.

25. Which of the following statements is true concerning an auditor’s responsibilities regarding financial
statements?
a. Making suggestions that are adopted about the form and content of an entity’s financial
statements impairs auditor’s independence.
b. An auditor may draft an entity’s financial statements based on information from management’s
accounting system.
c. The fair presentation of audited financial statements in conformity with GAAP is an implicit part of
the auditor’s responsibilities.
d. An auditor’s responsibilities for audited financial statements are not confined to the expression of
the auditor’s opinion.

26. Before accepting an engagement to audit a new client, an auditor is required to


a. Make inquiries of the predecessor auditor after obtaining the consent of the prospective client
b. Obtain the prospective client’s signature to the engagement letter
c. Prepare a memorandum setting forth the staffing requirements and documenting the preliminary
audit plan
d. Discuss the management representation letter with the prospective client’s audit committee

27. Which statement is correct relating to a potential successor auditor’s responsibility for
communicating with the predecessor auditors in connection with a prospective new audit client?
a. The successor auditors have no responsibility to contact the predecessor auditors
b. The successor auditors should obtain permission from the prospective client to contact the
predecessor auditors
c. The successor auditors should contact the predecessors regardless of whether the prospective
client authorizes contact
d. The successor auditors need not contact the predecessors if the successors are aware of all
available relevant facts
AT Quizzer 4 PSA 200 (Rev and Redrafted), 210 Redrafted, 220 Redrafted 4

28. An auditor who accepts an audit engagement and does not possess the industry expertise of the
business entity, should
a. Engage financial experts familiar with the nature of the business entity
b. Obtain a knowledge of matters that relate to the nature of the entity’s business
c. Refer a substantial portion of the audit to another cpa who will act as the principal auditor
d. First inform management that an unqualified opinion cannot be issued

29. Before accepting an audit engagement, a successor auditor should make specific inquiries of the
predecessor auditor regarding the predecessor’s
a. Awareness of the consistency in the application of generally accepted accounting principles
between periods
b. Evaluation of all matters of continuing accounting significance
c. Opinion of any subsequent events occurring since the predecessor’s audit report was issued
d. Understanding as to the reasons for the change of auditors

30. An auditor who has been invited to submit a proposal for an audit engagement is a
a. Predecessor auditor
b. Successor auditor
c. Principal auditor
d. Interim auditor

31. Lions Inc. has engaged the services of Mr. Reyes, CPA, to make a project study on the expanded
food vending operations of the corporation with the corresponding staffing and compensation
package for its executive staff. Reyes, however, has primarily auditing expertise and only in general
merchandising operations. Mr. Reyes may properly
a. Accept the engagement and carry it out consistent with generally accepted auditing standards
b. Accept the engagement but exercise due professional care
c. Accept the engagement and acquire the necessary competence or consult with established
authorities
d. Decline the engagement for lack of experience or competence in an entirely new line of
specialization

32. Which of the following statements about independent financial statement audit is correct?
a. The audit of financial statements relieves management of its responsibilities for the financial
statements.
b. An audit is designed to provide limited assurance that the financial statements taken as a whole
are free from material misstatement.
c. The procedures required to conduct an audit in accordance with psas should be determined by
the client who engaged the services of the auditor.
d. The auditor’s opinion is not an assurance as to the future viability of the entity as well as the
effectiveness and efficiency with which management has conducted the affairs of the entity.

33. The primary purpose of an independent financial statement audit is to


a. Provide a basis for assessing management’s performance
b. Comply with government regulatory requirements
c. Assure management that the financial statements are unbiased and free from material error
d. Provide users with an unbiased opinion about the fairness of information reported in the financial
statements

34. Financial statements need to be prepared in accordance with one or combination of:
Philippine Philippine Other AuthoMyrative or Philippine
Standards on Accounting Comprehensive Financial Financial
Auditing Standards Reporting Framework Reporting Standard
a. Yes yes yes yes
b. No yes yes yes
c. No yes no yes
d. Yes no no no

35. By providing high level of assurance on audit reports on financial statements, the auditor
a. Guarantees the fair presentation of the financial statements.
b. Confirms the accuracy of the financial statements.
c. Enhances the credibility of the financial statements.
d. Assures the readers that fraudulent activities of employees have been detected.
AT Quizzer 4 PSA 200 (Rev and Redrafted), 210 Redrafted, 220 Redrafted 5

36. The auditor may accept or continue an audit engagement only when the basis upon which it is to be
performed has been agreed, through
I. Establishing whether the preconditions for an audit are present
II. Confirming that there is a common understanding between the auditor and management, and
where appropriate, those charged with governance of the terms of the audit engagement
a. I only
b. II only
c. Both I and II
d. Neither I nor II

37. An audit is conducted on the premise that management and, where appropriate, those charged
with governance, have acknowledged and understand that they have responsibilities that are
fundamental to the conduct of an audit in accordance with PSAs. Which of the following is not one
of those responsibilities?
a. The preparation of financial statements in accordance with relevant pronouncements issued by
the AASC
b. The establishment and maintenance of an adequate internal control system that is necessary to
enable the preparation of financial statements that are free from material misstatement,
whether due to fraud or error
c. To provide the auditor with access to all information that is relevant to the preparation of the
financial statements such as records, documentation, and other matters
d. To provide the auditor with unrestricted access to persons within the entity from which the
auditor determines it necessary to obtain audit evidence

38. The auditor shall undertake which of the following activities prior to starting an initial audit?
I. Performing procedures required by PSA 220 (Quality Control for an Auditor of Financial Statements)
regarding the acceptance of the client relationship and the specific audit engagement
II. Communicating with the predecessor auditor, where there has been a change of auditors, in
compliance with relevant ethical requirements
a. I only
b. II only
c. Either I or II
d. Both I and II

39. The objective of the ordinary audit of financial statements is the expression of an opinion on:
a. The fairness of the financial statements in all material respects.
b. The accuracy of the financial statements.
c. The accuracy of the annual report.
d. The accuracy of the balance sheet and income statement.

40. Auditors accumulate evidence to:


a. Defend themselves in the event of a lawsuit.
b. Justify the conclusions they have otherwise reached.
c. Satisfy the requirements of the Securities and Exchange Commission.
d. Enable them to reach conclusions about the fairness of the financial statements.

**********************

PSA 210 Redrafted “Agreeing the Terms of Audit Engagements”

1. According to PSA 210, the auditor and the client should agree on the terms of engagement. The
agreed terms would need to be recorded in a(n)
a. Memorandum to be placed in the permanent section of the working papers.
b. Engagement letter.
c. Client representation letter.
d. Comfort letter.

2. 1st statement – The auditor and the client should agree on the terms of the engagement. Such an
agreement may be in the form of audit engagement letter or other suitable form of contract.

2nd statement – Even in those countries where the scope of the audit is established by law, an
engagement letter may be informative for the client.

a. 1st statement is true ; 2nd statement is false


b. 1st statement is false ; 2nd statement is true
c. Both statements are true.
d. Both statements are false.
AT Quizzer 4 PSA 200 (Rev and Redrafted), 210 Redrafted, 220 Redrafted 6

3. It documents and confirms the auditor’s acceptance of the appointment, the objective and scope of
the audit, the extent of the auditor’s responsibilities to the client and the form of any reports.
a. Representation letter
b. Management letter
c. Engagement letter
d. Comfort letter

4. Engagement letters are widely used in practice for professional engagements of all types. The
primary purpose of the engagement letter is to
a. Remind management that the primary responsibility for the financial statements rests with
management.
b. Satisfy the requirements of the CPA’s liability insurance policy.
c. Provide a starting point for the auditor’s preparation of the preliminary audit program.
d. Provide a written record of the agreement with the client as to services to be provided.

5. Which of the following is (are) valid reasons why an auditor sends to his client an engagement letter?
a b c d
To avoid misunderstanding with respect to management Yes Yes No No
To confirm the auditor’s acceptance of the appointment Yes Yes Yes No
To document the objective and scope of the audit Yes Yes Yes Yes
To ensure CPA’s compliance to PSA Yes No No Yes

6. In making arrangements for an audit, there should be a clear understanding between the auditor and
the client as to the following except:
a. The type of audit to be performed.
b. Terms of settlement for audit services.
c. Assurance of auditor’s independence.
d. Official to whom audit report shall be addressed.

7. An auditor who has been invited to submit a proposal for an audit engagement is a
a. Predecessor auditor.
b. Successor auditor.
c. Principal auditor.
d. Interim auditor.

8. Engagement letter that documents and confirms the auditor’s acceptance of the engagement would
normally be sent to the client.
a. Before the auditor report is issued.
b. After the audit report is issued.
c. At the end of the fieldwork.
d. Before the commencement of the engagement.

9. The nature of an audit engagement changes when there is


a. A change in circumstances.
b. A misunderstanding as to the nature of the audit.
c. A restriction on the scope of the engagement, whether imposed by management or caused by
circumstances.
d. All of the above
10. The form and content of the audit engagement letters may vary for each client, but they would
generally include reference to:
a. Management’s responsibility for all the financial statements
b. Restricted access to whatever records, documentation and other information requested in
connection with the audit.
c. The fact that because of the test nature and other inherent limitations of an audit, together with
the inherent limitations of any accounting and internal control system, there is an avoidable risk
that even some material misstatements may remain undiscovered.
d. The type of opinion that may be given out by the auditor.

11. The form and content of the audit engagement letters may vary for each client, but they would
generally include reference to except:
a. Management’s responsibility for all the financial statements.
b. The scope of the audit, excluding reference to applicable legislation, regulations, or
pronouncements of professional bodies to which the auditor adheres.
c. The form of any reports or any communication of results of engagement.
d. Unrestricted access to whatever records, documentation and other information requested in
connection with the audit.
AT Quizzer 4 PSA 200 (Rev and Redrafted), 210 Redrafted, 220 Redrafted 7

12. Which of the following matters is generally included in an auditor’s engagement letter?
a. Management’s responsibility for the entity’s compliance with laws and regulations.
b. The factors to be considered in setting preliminary judgments about materiality.
c. Management’s vicarious liability for illegal acts committed by its employees.
d. The auditor’s responsibility to search for significant internal control deficiencies.

13. 1st statement – On recurring audits, the auditor should consider whether circumstances require the
terms of the engagement to be revised and whether there is a need to remind the client of the existing
terms of engagement.

2nd statement – The auditor should send a new engagement letter each year to an established client.

a. 1st statement is true ; 2nd statement is false


b. 1st statement is false ; 2nd statement is true
c. Both statements are true.
d. Both statements are false.

14. On recurring audits, the auditor may decide not to send a new engagement letter each period. In
which of the following situations will there be no need to send a new letter?
a. Revision or special terms of the engagement.
b. Significant change in nature or size of the client’s business.
c. Recent change of middle management and rank and file organization structure.
d. Indications of misunderstanding of the objective and scope of the audit.

15. Which of the following will not necessarily lead the client to request for the auditor to change the
engagement to one which provides a lower level of assurance?
a. Restrictions on the scope of engagement, whether imposed by management or caused by
circumstances.
b. Misunderstanding as to the nature of the audit or related service originally requested.
c. Recent changes in senior management, board of directors or ownership.
d. Change in circumstances affecting the need for the service.

16. Examples of changes in the nature of an audit engagement include the following except:
a. A change in circumstances
b. A misunderstanding as to the nature of an audit
c. A restriction on the scope of the engagement, whether imposed by management or caused by
circumstances
d. None of the above

17. This is a gap that exists between what the public, especially users of financial statements, believe
auditors do (or ought to do) and what the auditors actually do is known as:
a. Auditor’s gap
b. User’s gap
c. Business gap
d. Expectation gap

18. 1st statement – Where the terms of the engagement are changed, the auditor and the client should
agree on the new terms.

2nd statement – The auditor should not agree to a change of engagement when there is no
reasonable justification for doing so.

3rd statement – If the auditor is unable to agree to a change of the engagement and is not permitted
to continue the original engagement, the auditor should withdraw and consider whether there is any
obligation, either contractual or otherwise, to report to other parties, such as the board of directors or
shareholders, the circumstances necessitating the withdrawal.

a. 1st and 2nd statements are correct; 3rd statement is incorrect.


b. 1st and 3rd statements are correct; 2nd statement is incorrect.
c. 2nd and 3rd statements are correct; 1st statement is incorrect.
d. All statements are correct.
AT Quizzer 4 PSA 200 (Rev and Redrafted), 210 Redrafted, 220 Redrafted 8

19. Which of the following factors do not influence the decision of the auditor to send a separate
engagement letter to the parent entity and its component (subsidiary, branch or division) assuming
the same auditor handles both entities?
a. Legal requirements
b. Degree of ownership by parent
c. Ethical requirements
d. Whether a separate audit report is to be issued on the component

20. The engagement letter will do one, some, or all of the following:
1. State whether the CPA will perform audit, review, or compilation services.
2. State whether the CPA will perform tax or management advisory or other services.
3. State any restriction to be imposed on the CPA’s work.
4. Identify deadlines for completing the work.
5. State the amount and type of work to be done by client’s personnel in generating auditor’s work
papers.
6. State the CPA’s fees for the engagement.
7. Inform the client that the CPA does not have responsibility for detecting fraud.

The engagement letter will do


a. Numbers 1, 2, 4 and 6.
b. Numbers 1, 2, 3 4 and 6.
c. Numbers 1, 3, 5 and 7.
d. All seven of the above stated items.

21. The engagement letter


a. Does affect the CPA firm’s responsibility to external users of audited financial statements.
b. Can affect legal responsibilities to the client.
c. Can be used to alter the auditor’s responsibilities under generally accepted auditing standards.
d. Is useful only if it is an engagement, but has no effect for review or compilation services.

22. Audit engagement letters generally include reference to the following except
a. Objective of the audit of financial statements.
b. Management’s responsibility for the financial statements.
c. Scope of the audit.
d. Identification of the audit team members.

23. When a professional accountant is the auditor of a parent entity and also the auditor of its subsidiary,
branch or division (component), which of the following factors need not be considered in deciding
whether to send the separate engagement letter to the component?
a. Who appoints the auditor of the component.
b. Whether a separate audit report is to be issued on the component.
c. Legal requirements.
d. Number of reports to be prepared during the peak audit season.

24. Which of the following would most likely cause a CPA not to accept an audit engagement?
a. Determining that the entity’s management lacks integrity.
b. The inability to perform audit procedures prior to year end.
c. Determining that the entity’s internal control is weak.
d. Appointment after the close of the fiscal year.

25. Which of the following helps prevent misunderstandings during audit planning?
a. Auditor involvement in the preparation of the client’s financial records.
b. Client involvement in determining specific audit planning issues.
c. A preliminary meeting conference with the client to discuss fees, timing, client assistance and
related issues.
d. Involvement of the client’s internal auditors in setting materiality levels and determining the
scope of audit tests.
26. The auditor’s understanding with a client as to the scope and nature of services to be provided is
usually documented in:
a. The preliminary planning memorandum.
b. The engagement letter.
c. Communications with the audit committee
d. The audit report
AT Quizzer 4 PSA 200 (Rev and Redrafted), 210 Redrafted, 220 Redrafted 9

27. A major difference of opinion concerning an accounting issue has risen between an assistant on an
audit engagement and the auditor with final responsibility for the engagement. If after the
consultation, the assistant believes it necessary to disassociate himself from the resolution of the
matter, both the auditor and his assistant must
a. Refer the matter to the firm’s peer review committee.
b. Inform management of the nature of disagreement.
c. Inform the company’s audit committee of the nature of disagreement.
d. Document the details of the disagreement and the basis of resolution.

28. Before accepting an audit engagement, a successor auditor should make specific inquiries of the
predecessor auditor regarding
a. The predecessor’s assessments of inherent risk and judgment about materiality.
b. The predecessor’s evaluation of matters of continuing accounting significance.
c. The degree of cooperation the predecessor received concerning the inquiry of client’s lawyer.
d. Disagreements the predecessor had with the client concerning auditing procedures and
accounting principles.

29. A successor auditor should request the new client to authorize the predecessor auditor to allow a
review of the predecessor’s
Engagement letter Working papers
a. Yes Yes
b. Yes No
c. No Yes
d. No No

30. Arrangements concerning with which of the following are least likely to be included in the
engagement letter?
a. A predecessor auditor
b. Fees and billing
c. CPA investment in client securities
d. Other services to be provided in addition to the audit

31. An engagement letter should ordinarily include information on the objectives of the engagement and
CPA’s Client’s Limitations of
Responsibilities Responsibilities Engagement
a. Yes Yes Yes
b. Yes No Yes
c. Yes No No
d. No No No

32. Before performing any audit procedures, the auditor and the client should agree on the
Type of opinion to be expressed Terms of engagement
a, Yes Yes
b. No Yes
c. Yes No
d. No No

33. Which of the following statements would least likely appear in an auditor’s engagement letter?
a. “Fees for our services are based on regular per diem rates, plus travel and other out-of-pocket
expenses.”
b. “During the course of our audit, we may observe opportunities for economy in, or improved
controls over your operations.”
c. “Our engagement is subject to the risk that material misstatements of fraud, if they exist, will not
be detected.”
d. “After performing our preliminary analytical procedures we will discuss with you the other
procedures we consider necessary to complete the engagement.”

34. Which of the following is not a key issue which need to agreed by the auditor and the client as part of
the “terms of engagement”?
a. Expectation gap
b. Issues of risk assessment
c. Use of experts
d. All of the above are key issues.
AT Quizzer 4 PSA 200 (Rev and Redrafted), 210 Redrafted, 220 Redrafted 10

35. An engagement letter would not normally include


a. Billing arrangements.
b. Arrangements concerning client’s assistance.
c. Details of procedures that will be performed.
d. Expectation of receiving a representation letter from management.

36. Audit engagement letters generally include reference to the following except
a. Objective of the audit of financial statements
b. Management’s responsibility for the financial statements
c. Scope of the audit
d. Identification of the audit team members

37. When a professional accountant is the auditor of a parent entity and also the auditor of its
subsidiary, branch or division (component), which of the following factors need not be considered in
deciding whether to send the separate engagement letter to the component?
a. Who appoints the auditor of the component
b. Whether a separate audit report is to be issued on the component
c. Legal requirements
d. Number of reports to be prepared during the peak audit season

38. On recurring audits, the auditor may decide not to send a new engagement letter each period. In which
of the following situations will there be no need to send a new letter?
a. Revision or special terms of the engagement
b. Significant change in nature or size of the client’s business
c. Recent change of middle management and rank and file organization structure
d. Indications of misunderstanding of the objective and scope of the audit

39. Which of the following will not necessarily lead the client to request for the auditor to change
the engagement to one which provides a lower level of assurance?
a. Restrictions on the scope of the engagement, whether imposed by management or caused by
circumstances
b. Misunderstanding as to the nature of an audit or related service originally requested
c. Recent changes in senior management, board of directors or ownership
d. Change in circumstances affecting the need for the service

40. For initial engagements, the auditor should obtain sufficient appropriate audit evidence for the
following, except
a. That the opening balances do not contain misstatement that materiality affect the current
period’s financial statements
b. That the prior period’s closing balances have been correctly brought forward to the current period,
or, when appropriate, have been restated
c. That appropriate accounting policies are consistently applied or changes in accounting policies
have been properly accounted for or adequately disclosed
d. That the client has informed the predecessor auditor of his appointment as the new auditor

************************************

Quizzer – PSA 220 Redrafted “Quality Control for an Audit of F/S”

1. A CPA establishes quality control policies and procedures for deciding whether to accept a new client
or continue to perform services for a current client. The primary purpose for establishing such policies
and procedures is
a. To lessen the exposure to litigation resulting from failure to detect irregularities in client’s financial
statements.
b. To enable the auditor to attest to the integrity or reliability of a client.
c. To comply with the quality control standards established by regulatory bodies.
d. To minimize the likelihood of association with clients whose management lacks integrity.
AT Quizzer 4 PSA 200 (Rev and Redrafted), 210 Redrafted, 220 Redrafted 11

2. The following factors affect the nature, timing and extent of an audit firm’s quality control policies and
procedures, except:
a b c d

• Size and nature of practice Yes Yes No No


• Geographic dispersion Yes Yes Yes No
• Organization Yes No Yes No
• Appropriate cost/benefit
Considerations Yes Yes No No

3. Audit firms are subject to many risks when they carry out an audit assignment. As a result of this, the
firm may suffer:
a. Action through the courts so that damages are payable.
b. Loss of client.
c. Adverse publicity and loss of reputation.
d. All of the following may be suffered by the firm

4. Which of the following statements is incorrect?


a. Quality controls are the policies and procedures adopted by a firm in all its audit work.
b. Personnel of an auditing firm are presumed to know all the firm’s general quality control policies
and procedures and need not be communicated to them whenever they are sent out for audit
engagements.
c. Quality control policies are objectives and goals, while quality control procedures are steps to be
taken to accomplish the policies adopted.
d. The examination by CPAs of a CPA firm’s auditing practices to ascertain compliance with its
quality control system is known as a peer review.

5. Risk audit firms are subject to when they carry out an audit assignment.
a. Giving an unqualified report when a qualified one was required.
b. Qualifying a report unnecessarily.
c. Being negligent such that loss is caused to the client or third party.
d. All of the above

6. 1st statement – It is in the interests of the firm to manage the firm overall and each individual audit
such that the Code of Professional Ethics for CPAs is followed at all times.

2nd statement – It is in the interests of the firm to manage the firm overall and each individual audit
such that the Philippine Auditing Standards (PSAs) are followed at all times.

a. 1st statement is true ; 2nd statement is false


b. 1st statement is false; 2nd statement is true
c. both statements are true
d. both statements are false

7. Factors to consider when designing quality control procedures include all of the following except
a. Size of the practice
b. Organization of the practice
c. Cost-benefit considerations
d. None of the following

8. Quality control in audit firms, both at the firm level and at the level of a specific audit is essential to all
audit firms. Methods of quality control in large firms include all of the following except
a. Peer reviews
b. Use of technical manuals
c. Sub-contracting work to other practices
d. Training

9. A review undertaken by a more experienced member of the audit firm to ensure that the work has
been performed in accordance with the audit plan and that the conclusions expressed are consistent
with the results of the works performed:
a. Peer review
b. Post audit review
c. “Hot” review
d. Audit review
AT Quizzer 4 PSA 200 (Rev and Redrafted), 210 Redrafted, 220 Redrafted 12

10. A CPA firm should establish procedures for conducting and supervising work at all organizational
levels to provide reasonable assurance that the work performed meets the firm’s standards of quality.
To achieve this goal, the firm most likely would establish procedures for
a. Evaluating prospective and continuing client relationships.
b. Reviewing engagement working papers and reports.
c. Requiring personnel to adhere to the applicable independence rules.
d. Maintaining personnel files containing documentation related to the evaluation of personnel.

11. The primary reason why a CPA firm establishes quality control policies and procedures for
professional development of staff accountants is to
a. Comply with the continuing educational requirements imposed by various states for all staff
accountants in CPA firms.
b. Establish, in fact as well as appearance, that staff accountants are increasing their knowledge of
accounting and auditing matters.
c. Provide a forum for staff accountants to exchange their experiences and views concerning firm
policies and procedures.
d. Provide reasonable assurance that staff personnel will have the knowledge required to enable
them to fulfill responsibilities.

12. A firm of CPAs may use policies and procedures such as notifying professional personnel as to the
names of audit clients having publicly held securities and confirming periodically with such personnel
that prohibited relations do not exist. This is done to achieve effective quality control in which of the
following areas?
a. Acceptance and continuance of clients.
b. Assigning personnel to engagement.
c. Independence.
d. Inspection.

13. In pursuing a CPA firm’s quality control objectives, a CPA firm may maintain records indicating which
partners or employees of the CPA firm were previously employed by the CPA firm’s clients. Which
quality control objective would this be most likely to satisfy?
a. Professional relationship.
b. Supervision.
c. Independence.
d. Advancement.

14. Which of the following is not an example of quality control procedure likely to be used by a public
accounting firm to meet its professional responsibilities to clients?
a. Completion of independence questionnaires by all partners and employees.
b. Review and approval of audit plan by the partner in charge of the engagement just prior to signing
the auditor’s report.
c. Evaluating professional staff after the conclusion of each engagement.
d. Evaluating its integrity of management for each new audit client.

15. Which of the following statements regarding quality control policies and procedures is incorrect?
a. Quality control policies and procedures should be implemented at both the level of the audit firm
and on an individual audit.
b. The audit firm should implement quality control policies and procedures designed to ensure that
all audits are conducted in accordance with PSAs or relevant national standards or practices.
c. Quality control policies are objectives and goals while quality control procedures are steps to be
taken to accomplish the policies adopted.
d. PSA 220 prescribes all the procedures to implement each quality control policy applicable to
auditing firms.

16. A CPA firm evaluates its personnel advancement experience to ascertain whether individuals meeting
stated criteria are assigned increased degrees of responsibility. This is evidence of the firm’s
adherence to which of the following prescribed standards:
a. Professional ethics.
b. Supervision and review.
c. Accounting and review services.
d. Quality control.
AT Quizzer 4 PSA 200 (Rev and Redrafted), 210 Redrafted, 220 Redrafted 13

17. A CPA firm’s personnel partner periodically studies the CPA firm’s personnel advancement
experience to ascertain whether individuals increased degrees of responsibility. This is evidence of
the CPA firm’s adherence to prescribed standards of
a. Quality control.
b. Due professional care.
c. Supervision and review.
d. Fieldwork.

18. ABC & Co., a large international CPA firm, is to have an “external peer review”. The peer review will
most likely be performed by
a. Employees and partners of ABC & Co. who are not associated with the particular audits being
reviewed.
b. Audit review staff of the Securities and Exchange Commission.
c. Audit review staff of the PICPA.
d. Employees and partners of another CPA firm.

19. The objective of quality control mandates that a public accounting firm should establish policies and
procedures for professional development which provide reasonable assurance that all entry-level
personnel
a. Prepare working papers which are standardized in form and content.
b. Have the knowledge required to enable them to fulfill responsibilities assigned.
c. Will advance within the organization.
d. Develop specialties in specific areas of public accounting.

20. The nature and extent of a CPA firm’s quality control policies and procedures depend on:
CPA firm’s size Nature of CPA Cost-benefit
firm’s practice considerations
a. Yes Yes Yes
b. Yes Yes No
c. Yes No Yes
d. No Yes Yes

21. A basic objective of a CPA firm is to provide professional services to conform with professional
standards. Reasonable assurance of achieving this basic objective is provided through
a. Continuing professional education
b. System of quality control
c. Compliance with generally accepted reporting standards.
d. System of peer review.

22. A CPA firm evaluates its personnel advancement experience to ascertain whether individuals meeting
stated criteria are assigned increased degrees of responsibility. This is evidence of the firm’s
adherence to which of the following prescribed standards:
a. Professional ethics.
b. Supervision and review.
c. Accounting and review services.
d. Quality control.

23. The work performed by each assistant should be reviewed to determine whether it was adequately
performed and to evaluate whether the
a. Auditor’s system of quality control has been maintained at a high level.
b. Results are consistent with the conclusions to be presented in the auditor’s report.
c. Audit procedures performed are approved in the professional standards.
d. Audit has been performed by persons who have adequate technical training and proficiency as
auditors.

24. In pursuing quality control objectives with respect to acceptance of a client, a CPA firm is likely to
a. Make inquiries of the proposed client’s legal counsel.
b. Review financial statements of the proposed client.
c. Make inquiries of previous auditors.
d. Review the personnel practices of the proposed client.

25. Within the context of quality control, the primary purpose of continuing professional education and
training activities, is to enable a CPA firm to provide personnel within the firm with
a. Technical training that assures proficiency as an auditor.
b. Professional education that is required in order to perform with due professional care.
c. Knowledge required to fulfill assigned responsibilities and to progress within the firm.
d. Knowledge required in order to perform a peer review.
AT Quizzer 4 PSA 200 (Rev and Redrafted), 210 Redrafted, 220 Redrafted 14

26. A process comprising an ongoing consideration and evaluation of the firm’s system of quality
control, including a periodic inspection of a selection of completed engagements, designed to enable
the firm to obtain reasonable assurance that its system of quality control is operating effectively.
a. Inspection
b. Engagement quality control reviewer
c. Quality review
d. Monitoring

27. In relation to completed engagements, these are procedures designed to provide evidence
of compliance by engagement teams with firm’s quality control policies and procedures.
a. Inspection
b. Quality control
c. Monitoring
d. Engagement quality control reviewer

28. The inspection of a selection of completed engagements is ordinarily performed on a cyclical


basis. Engagements selected for inspection include at least:
a. One engagement for each engagement partner over an inspection cycle
b. Two engagements for each engagement partner over an inspection cycle
c. One engagement for each firm over an inspection cycle
d. Two engagements for each firm over an inspection cycle

29. An inspection cycle ordinarily spans no more than:


a. One year
b. Two years
c. Three years
d. Four years

30. FHV and Company has just completed its annual independence review. Francis, the partner in-
charge for independence, is reviewing a completed copy of the independence declaration form and
noticed that Miss Myra, an audit manager declared that her husband was newly hired as a controller
of a Company where Miss Myra is the engagement manager. As partner-in-charge for independence
what is the best course of action to take in the situation?
a. Discuss the matter with Miss Myra and the engagement partner and ensure that all work done by
Miss Myra is properly reviewed by the engagement partner.
b. Assigned an Engagement Quality Control Reviewer to ensure that all judgment made by Miss
Myra are reviewed and ensure she remains objective.
c. Transfer Miss Myra to another engagement to reduce to risk of non-compliance with the
independence requirement.
d. Discuss the matter with management and those charged with governance and if they agree to
continue with Miss Myra as audit manager, then should continue managing the engagement.

31. In performing an acceptance and continuance procedures for a newly accepted engagement, Justine,
a new partner in the Firm should obtain information relating to:
I. The integrity of the clients management
II. Independence of the firm
III. Competency to serve the client appropriately
a. I and II only
b. II and III only
c. I and III only
d. I, II, III

32. Who is responsible for forming a conclusion on compliance with independence requirements
that apply to the audit engagements?
a. Audit staff
b. Audit supervisor
c. Engagement manager
d. Engagement partner

33. The firm should communicate the results of the monitoring of its quality control system to
engagement partners and other appropriate individuals within the firm at least:
a. Every six months
b. Every twelve months
c. Every two years
d. Every three years
AT Quizzer 4 PSA 200 (Rev and Redrafted), 210 Redrafted, 220 Redrafted 15

34. The firm should establish policies and procedures requiring appropriate documentation to provide
evidence of the operation of each element of its system of quality control. How such matters are
documented is based on:
a. PSQC1
b. PSA 220
c. Philippine Audit Practice Statements
d. The firm’s decision

35. A firm has obtained information that would have caused it to decline an engagement had the
information been available earlier. Actions available to the auditor include the following, except:
a. Reporting the information and its implications to the person(s) who appointed the CPA
b. Withdraw from the engagement
c. Withdraw from both the engagement and the client relationship
d. Continue the engagement, since the Code of Ethics requires started engagements to be
finished regardless of subsequent developments and information

36. The following methods are most likely to develop capabilities and competence, except:
a. Professional education
b. Continuing professional development, including training
c. Work experience and coaching by less experienced staff
d. Self-study modules on professional accounting and auditing literature

37. Supervision includes:


a. Tracking the progress of the engagement
b. Considering the capabilities and competence of individual members of client personnel
c. Addressing all issues arising during the engagement and modifying the planned approach
appropriately
d. Identifying maters for consultation or consideration by less experienced engagement team
members during the engagement

38. Review responsibilities are determined on the basis that ______ experienced team members,
review the work of ______ experienced team members
a. more, more
b. more, less
c. less, more
d. less, less

39. An engagement quality control review is required to be performed:


a. Immediately after the re-assessment of control risk
b. For all audits of financial statements of listed entities
c. For all types of audits, regardless of the subject matter of the engagement
d. At engagement completion after the report is issued

40. What aspects are most important in determining the eligibility of engagement quality
control reviewers?
a. Technical qualifications and objectivity
b. Integrity and objectivity
c. Competence and independence
d. All of these

*********************************
Auditing Theory AT - Quizzer 5
Advanced Review Solutions

“Audit Documentation”

1. According to PSA 230 Redrafted “Documentation”, working papers do not


a. Assist in the planning and performance of the audit.
b. Assist in the supervision and review of the audit work.
c. Record the audit evidence resulting from the audit work performed to support an auditor’s
opinion.
d. Support the client’s financial statements..

2. 1st statement – Working papers are the property of the auditor.


2nd statement – Although portions of or extracts from the working papers maybe made available to
the entity at the discretion of the auditor, they may be substitute for the entity’s accounting records.
a. 1st statement is True ; 2nd statement is False
b. 1st statement is False; 2nd statement is True
c. Both statements are True
d. Both statements are False

3. 1st statement – The auditor should prepare working papers which are sufficiently complete and
detailed to provide an overall understanding of the audit.
2nd statement – The auditor should record in the working papers information on planning the audit
work, the nature, timing and extent of the audit procedures performed, the results thereof, and the
conclusions drawn from the audit evidence obtained.
a. 1st statement is True ; 2nd statement is False
b. 1st statement is False ; 2nd statement is True
c. Both statements are True
d. Both statements are False

4. Why does an auditor document audit evidence?


a. To comply with the requirements of gathering all available evidence.
b. To provide client reference for all account balances and correcting entries.
c. To support audit opinion and to provide evidence that the audit was carried out in accordance
with PSA.
d. To document all records of misstatements noted in the financial statements.

5. Working papers that record the procedures used by the auditor to gather evidence should be
a. Considered the primary support for the financial statements being audited.
b. Viewed as the connecting link between the books of accounts and the financial statements.
c. Designed to meet the circumstances of the particular engagement.
d. Destroyed when the audited entity ceases to be a client.

6. Which of the following conditions constitutes inappropriate working paper preparation?


a. Flowcharts are included in the working papers.
b. Findings are cross-referenced to supporting documentation.
c. Tick marks are explained in working papers.
d. All forms and directives used by the auditee department are included in the working papers.

7. An auditor’s working papers will ordinarily be least likely to include documentation showing how the
a. Client’s schedules were prepared.
b. Engagement was planned.
c. Understanding of the client’s internal control was obtained and control risk was assessed.
d. Unusual matters were resolved.

8. Which of the following is usually included or shown in the auditor’s working papers?
a. The procedures used by the auditor to verify the personal financial status of members of the
client’s management team
b. Analyses that are designed to be a part of, or a substitute for, the client’s accounting records
c. Excerpts from authoritative pronouncements that support the underlying generally accepted
accounting principles used in preparing the financial statements
d. The manner in which exceptions and unusual matters disclosed by the auditor’s procedures were
resolved or treated
AT - Quizzer 5 “Audit Documentation” 2

9. Which of the following is not generally included in the working papers file?
a. An indication as to who performed the audit procedures and when they were performed.
b. Documentation of the auditor’s understanding of the accounting and internal control systems.
c. Copy of the internal audit’s audit program.
d. Analyses of significant ratios and trends.

10. Although the quantity and content of audit working papers vary with each particular engagement, an
auditor’s permanent files most likely include
a. Schedules that support the current year’s adjusting entries.
b. Prior years’ accounts receivable confirmations that were classified as exceptions.
c. Documentation indicating that the audit work was adequately planned and supervised.
d. Analyses of share capital and other owners’ equity accounts.

11. Audit working papers are indexed by means of reference numbers. The primary purpose of indexing
is to
a. Permit cross-referencing and simplify supervisory review.
b. Support the audit report.
c. Eliminate the need for follow-up reviews.
d. Determine that working papers adequately support findings, conclusions, and reports.

12. The audit working paper that reflects the major components of an amount reported in the financial
statements is the
a. Interbank transfer schedule c. Supporting schedule
b. Carryforward schedule d. Lead schedule

13. An auditor ordinarily uses a working trial balance resembling the financial statements without
footnotes, but containing columns for
a. Cash flow increases and decreases c. Reclassifications and adjustments
b. Audit objectives and assertions d. Reconciliations and tickmarks

14. In the course of the audit of financial statements for the purpose of expressing an opinion thereon, the
auditor will normally prepare a schedule of unadjusted differences for which he did not propose
adjustment when they were uncovered. The primary purpose served by this schedule is to
a. Point out to the responsible client officials the errors made by various company personnel.
b. Summarize the adjustments that must be made before the company can prepare and submit its
income tax returns.
c. Identify the potential financial statement effects of errors or disputed items that were considered
immaterial when discovered.
d. Summarize the errors made by the company so that corrections can be made after the audited
financial statements are released.

15. Which of the following analyses appearing in a predecessor’s working papers is the successor auditor
least likely to be interested in reviewing?
a. Analysis of noncurrent balance sheet accounts. c. Analysis of income statement accounts.
b. Analysis of current balance sheet accounts. d. Analysis of contingencies.

16. In an internal audit, the audit supervisor determines that working papers are complete
a. When satisfied that the audit objectives have been met and the working papers support the
conclusions.
b. When working papers refer to the steps outlined in the audit program.
c. Only after the auditor who prepared the working papers has signed and dated them.
d. When proper cross-references to other working papers are noted.

17. Standardized working papers are often used, chiefly because they allow working papers to be
prepared more
a. Efficiently b. Professionally c. Neatly d. Accurately

18. Ordinarily, the audit may use schedules, analyses and other documentation prepared by entity
personnel in order to:
a. Lessen auditor’s responsibility.
b. Eliminate the need to apply any audit procedures on verifying their correctness.
c. Emphasize that the responsibility on financial statements rests with the client management.
d. Improve audit efficiency.
19. Working papers which contain information relating primarily to the audit of a single period.

a. Current audit files c. Financial reporting files b. Permanent audit files d.


Correspondence files
AT - Quizzer 5 “Audit Documentation” 3

20. Which of the following is the least required of the audit working papers?
a. Substitute for the entity’s accounting records.
b. Confidentiality of information included in the working papers.
c. Safe custody of the working papers.
d. Retention for a period sufficient to meet the needs of the practice.

21. The form and content of working papers are affected by matters such as the following except:
a. Nature of the engagement
b. Type of opinion to be rendered by the auditor
c. Nature and complexity of the business
d. Needs in the particular circumstances for the direction, supervision and review of work performed
by assistants.

22. Audit working papers should not


a. Include any client-prepared papers or documents other than those prepared by the CPA or his
assistant.
b. Be kept by the CPA after review and completion of the audit except for items required for the
income tax return or the permanent file.
c. Be submitted to the client to support the financial statements and to provide evidence of the audit
work performed.
d. Be themselves be expected to provide sufficient support for the auditor’s opinion.

23. Using laptop computers in auditing may affect the methods used to review the work of staff assistants
because
a. Supervisory personnel may not have an understanding of the capabilities and limitations of
computers.
b. Working paper documentation may not contain readily observable details of calculations.
c. The audit fieldwork standards for supervision may differ.
d. Documenting the supervisory review may require assistance of management services personnel.

24. The current file of the auditor’s working papers generally should include
a. A flowchart of the internal controls. c. A copy of the financial statements b. Copies of
bond and note indentures. d. Organization charts

25. Which of the following eliminates voluminous details from the auditor’s working trial balance by
classifying and summarizing similar or related items?
a. Account analyses
b. Supporting schedules
c. Control accounts
d. Lead schedules

26. Documentation is a form of evidence


a. Used in every financial statement audit.
b. Used in most financial statement audit.
c. Used on the rare occasions when it is both readily available and less costly than other
procedures.
d. Used when nothing is available that is more competent

27. An audit working paper that shows the detailed evidence and procedures regarding the balance in the
accumulated depreciation account in the year under audit will be found in the
a. Current file of working papers.
b. Permanent file of working papers.
c. Other information working papers in the current file.
d. Planning memorandum in the current file.

28. In the case of recurring audits, some working papers files may be classified as audit files which are
updated with new information of continuing importance. This type of audit file is known as:
a. Current audit file. c. Electronic audit file.
b. Permanent audit file. d. Planning memorandum file

29. A schedule listing account balances for the current and previous years, and columns for adjusting and
reclassifying entries proposed by the auditors to arrive at the final amount that will appear in the
financial statements, is referred to as a
a. Working trial balance. c. Summarizing schedule
b. Lead schedule. d. Supporting schedule

30. A schedule set up to combine similar general ledger accounts, the total of which appears on the
working trial balance as a single amount, is referred to as a
AT - Quizzer 5 “Audit Documentation” 4

a. Supporting schedule. c. Corroborating schedule


b. Lead schedule. d. Reconciling schedule

31. What do you call the type of working paper where matters of importance are noted down for further
verification?
a. Summary sheet. c. Agenda sheet
b. Audit program. d. Supporting schedules

32. The permanent file portion of the auditor’s working papers generally should include
a. A copy of the engagement letter.
b. A copy of key customer confirmation.
c. Names and addresses of audit staff personnel on the engagements.
d. Time and expense reports.

33. The permanent file section of the working papers that is kept for each audit client most likely contains
a. Review notes pertaining to questions and comments regarding the audit work performed.
b. A schedule of time spent on the engagement by each individual auditor.
c. Correspondence with the client’s legal counsel concerning pending litigation.
d. Narrative descriptions of the client’s accounting procedures and internal controls.

34. In general, which of the following statements is correct with respect to ownership, possession, or
access to working papers prepared by a CPA firm in connection with an audit?
a. The working papers may be obtained by third parties where they appear to be relevant to issues
raised in litigation.
b. The working papers are subject to the privileged communication rule which, in a majority of
jurisdictions, prevents third-party access to the working papers.
c. The working papers are the property of the client after the client pays the fees.
d. The working papers must be retained by the CPA firm for a period of ten years.

35. For what minimum period should audit working papers be retained by the independent CPA?
a. For the period during which the entity remains a client of the independent CPA.
b. For the period during which an auditor-client relationship exists but not more than six (6) years.
c. For the statutory period within which legal action may be brought against the independent CPA.
d. For as long as the CPA is in public practice.

36. Which of the following documentation is required for an audit in accordance with PSA?
a. An internal control questionnaire.
b. A client engagement letter.
c. A planning memorandum or checklist.
d. A client representation letter.

37 Audit documentation is the record of audit procedures performed, relevant audit evidence, and the
auditor’s conclusions. Which of the following statements concerning audit documentation is incorrect?

a. Audit documentation should include superseded drafts of working papers and financial
statements.
b. Audit documentation prepared after the performance of the audit work is likely to be less accurate
than documentation prepared at the time such work is performed.
c. Audit documentation may include abstracts or copies of the entity’s records such as significant
and specific contracts and agreements
d. Audit documentation is not a substitute for the entity’s accounting records.

38 The auditor is required to complete the administrative process of assembling the final audit file on a
timely basis after the date of the auditor’s report. The time limit within which to complete the assembly
of the audit file is ordinarily
a. Not more than 30 days after the date of the auditor’s report.
b. Not more than 60 days after the date of the auditor’s report.
c. Not more than 90 days after the end of the entity’s reporting period.
d. Not more than 60 days after the date the entity’s financial statements are authorized for issue.

39. Audit documentation (working papers) should


a. Be considered the primary support for the financial statements being audited.
b. Show that the accounting records agree or reconcile with the financial statements.
c. Evaluate management’s performance against company goals for the year under audit.
d. Be provided to the client upon request.

40. Audit documentation (working papers)


a. Provides evidence of the auditor's compliance with generally accepted auditing standards.
AT - Quizzer 5 “Audit Documentation” 5

b. Is a record to be used as a basis for the following year's engagement.


c. Includes an analysis of the client’s marketing strategies.
d. Is a client-owned record of conclusions reached by the auditors.

41. In using the work of a specialist, an understanding should exist among the auditor, the client, and
the specialist as to the nature of the work to be performed by the specialist. Preferably, the
understanding should be documented and would include all of the following except
a. The objectives and scope of the specialist’s work
b. The specialist’s representations as to his relationship, if any, to the client
c. The specialist’s understanding of the auditor’s corroborative use of the specialist’s findings in
relation to the representations in the financial statements
d. A statement that the methods or assumptions to be used are not inconsistent with those used
by the client

42. Which of the following is an invalid description of why working papers are developed?
a. Facilitates third-party reviews
b. Aids in the planning, performance and review of audits
c. Provides the principal evidential support for the auditor’s report
d. Aids in the professional development of the operating staff

43. The main advantage of property indexed working papers is to


a. Reduce the size of the file
b. Better organize the working papers
c. Allow division of labor within the audit team
d. Facilitate the efficient use of audit staff

44. Which of the following statements about working papers is correct?


a. Working papers are not permitted to be used as a reference source by the client
b. The auditor should document his understanding of the client’s internal control which is to be
used to plan the audit
c. Working papers may be regarded as a substitute for the client’s accounting records
d. When reporting on comparative financial statements, the independent auditor may discard
working papers after two years

45. Which of the following is a basic tool used by the auditor to control the audit work and review the
progress of the audit?
a. Time and expense summary
b. Engagement letter
c. Progress flowcharts
d. Audit program

46. During the working paper review, an audit supervisor finds that the auditor’s reported findings are
not adequately cross-referenced to supporting documentation. The supervisor will most likely
instruct the auditor to
a. Prepare a working paper to indicate that the full scope of the audit was carried out
b. Familiarize himself with the sequence of working papers so that he will be able to answer
questions about the conclusions stated in the report
c. Eliminate any cross-references to other working papers since the system is unclear
d. Provide a working paper indexing system that shows the relationship between findings,
conclusions, and the related facts

47. Which of the following working papers would one normally expect to find in the permanent file?
a. A copy of a long-term bond indenture
b. The working trial balance
c. An analysis of additions and disposals relating to marketable securities
d. A workpaper analyzing customer replies to confirmation requests

48. The permanent file section of the working papers that is kept for each audit client most likely
contains
a. Review notes pertaining to questions and comments regarding the audit work performed
b. A schedule of time spent on the engagement by each individual auditor
c. Correspondence with the client’s legal counsel concerning pending litigation
d. Narrative descriptions of the client’s internal control policies and procedures

49. Which of the following statements is correct with respect to ownership of audit
documentation? a. The audit firm owns the audit documentation
AT - Quizzer 5 “Audit Documentation” 6

b. The audit client owns the audit documentation


c. The audit client and audit firm have joint ownership of the audit documentation
d. The law is not explicit with respect to the ownership of audit documentation

50. After the fieldwork audit procedures are completed, a partner of the CPA firm who has not been
involved in the audit performs a second or wrap-up working paper review. This second review
actually focuses on
a. The fair presentation of the financial statements in conformity with applicable reporting
framework.
b. Fraud involving the client’s management and its employees
c. The materiality of the adjusting entries proposed by the audit staff
d. The communication of internal control weaknesses to the client’s audit committee

**************************************
Auditing Theory AT Quizzer 6
Advanced Review Solutions

Consideration of Fraud in an Audit of FS

1. Fraud include all of the following except:


a. recording of transactions without substance
b. suppression or omission of the effects of transactions from records or documents
c. mathematical or clerical mistakes in the underlying records and accounting data.
d. misappropriation of assets

2. Error include all of the following except:


a. misapplication of accounting policies
b. manipulation, falsification or alteration of records and documents.
c. mathematical or clerical mistakes in the underlying records and accounting data
d. oversight of accounting policies

3. 1st statement – The responsibility for the prevention and detection of fraud and error rests with
the auditor through implementation of accounting and internal control systems.
2nd statement – The accounting and internal control systems eliminate the possibility of fraud and
error.
a. 1st statement is True ; 2nd statement is False
b. 1st statement is False ; 2nd statement is True
c. Both statements are True
d. Both statements are False

4. 1st statement – The auditor is not and cannot be held responsible for the prevention of fraud and
error.
2nd statement – Annual audits may be carried out which may not; however, act as deterrent.
a. 1st statement is True ; 2nd statement is False
b. 1st statement is False ; 2nd statement is True
c. Both statements are True
d. Both statements are False

5. Under PSA 240, which of the following would be classified as an error?


a. Misappropriation of assets for the benefit of management.
b. Misinterpretation by management of facts that existed when the financial statements
were prepared.
c. Preparation of records by employees to cover a fraudulent scheme.
d. Intentional omission of the recording of a transaction to benefit a third party.

6. The types of intentional misstatements that are relevant to the auditor’s consideration of fraud
include
I. Misstatements resulting from fraudulent financial reporting
II. Misstatements resulting from misappropriation of assets.
a. I and II
b. I only
c. II only
d. Neither I nor II

7. The primary responsibility for the prevention and diction of fraud and error rests with
a. the auditor
b. those charged with governance
c. the management of the entity
d. both b and c

8. According to PSA 250, “Consideration of Laws and Regulations in an Audit of Financial


Statements”, the following are indications that noncompliance may have occurred, except
a. Investigation by government departments or payment of fines or penalties.
b. Management is dominated by one person or a small group and there is no effective
oversight board or committee.
c. Unauthorized transactions or improperly recorded transactions.
d. Purchasing at prices significantly above or below market price.
AT Quizzer 6 Consideration of Fraud in Audit of FS 2

9. This refers to acts of omission or commission by the entity being audited, either intentional or
unintentional which are contrary to the prevailing laws and regulations.
a. Error
b. Fraud
c. Noncompliance
d. Defalcation

10. An auditor who discovers that client employees have committed an illegal act that has a material
effect on the client’s financial statements, most likely would withdraw from the engagement if
a. The illegal act is a violation of generally accepted accounting principles.
b. The client does not take the remedial action that the auditor considers necessary.
c. The illegal act was committed during a prior year that was not audited.
d. The auditor has already assessed control risk at the maximum level.

11. Which of the following statements concerning illegal acts by clients is correct?
a. An auditor’s responsibility to detect illegal acts that have a direct and material effect on
the financial statements is the same as that for errors and irregularities.
b. An audit in accordance with generally accepted auditing standards normally includes
audit procedures specifically designed to detect illegal acts that have an indirect but
material effect on the financial statements.
c. An auditor considers illegal acts from the perspective of the reliability of management’s
representation rather than their relation to audit objectives derived from financial
statement assertions.
d. An auditor has no responsibility to detect illegal acts by clients that have an indirect effect
on the financial statements.

12. Which of the following is least likely a category of fraud risk factors that relate to misstatements,
resulting from fraudulent financial reporting?
a. Management’s characteristics and influence over the control environment.
b. Industry conditions
c. Operating characteristics and financial stability
d. Susceptibility of assets to misappropriation

13. In comparing management fraud with employee fraud, the auditor’s risk of failing to discover the
fraud is
a. Greater for employee fraud because of the higher crime rate among blue collar workers.
b. Greater for management fraud because of management’s ability to override existing
internal controls.
c. Greater for employee fraud because of the larger number of employees in the
organization.
d. Greater for management fraud because managers are inherently smarter than
employees.

14. Whether the auditor has performed an audit in accordance with PSA is determined by
a. The adequacy of the audit procedures performed in the circumstances and the suitability
of the auditor’s report based on the result of these procedures.
b. The absence of material misstatements
c. The absence of material errors
d. The Securities and Exchange Commission

15. 1st Statement – The auditor should communicate audit matters of governance interest arising
from the audit of financial statements with those charged with governance of an entity.

2nd statement – The auditor should determine the relevant persons who are charged with
governance and with whom audit matters of governance interest are not communicated.
a. 1st statement is True ; 2nd statement is False
b. 1st statement is False ; 2nd statement is True
c. Both statements are True
d. Both statements are False

16. Under PSA 260, “Communications of Audit Matters With Those Charged with Governance”, the
effectiveness of communications is enhanced by developing a constructive working relationship
between the auditor and those charged with governance. This relationship is developed while
maintaining an attitude of
a. professional independence and objectivity
b. loyalty and objectivity
c. professional independence and confidentiality
d. objectivity and confidentiality
AT Quizzer 6 Consideration of Fraud in Audit of FS 3

17. The auditor’s communication with those charged with governance may be made orally or in
writing. The auditor’s decision whether to communicate orally or in writing is affected by factors
except:
a. The nature, sensitivity and significance of the audit matters of governance interest to be
communicated.
b. The arrangements made with respect to periodic meetings or reporting of audit matters of
governance interest.
c. The size, operating structure, legal structure and communications processes of the entity
being audited.
d. The amount of past contact and dialogue the auditor has with those charged with
governance.

Other References

18. Which of the following best describes what is meant by the term “fraud risk factor”?
a. Factors whose presence indicates that the risk of fraud is high.
b. Factors whose presence often has been observed in circumstances where frauds have
occurred.
c. Factors whose presence requires modifications of planned audit procedures.
d. Reportable conditions identified during audit.

19. When conducting an audit, errors that arouse suspicion of fraud should be given greater attention
than other errors. This is an example of applying the criterion of
a. reliability of evidence
b. materiality
c. risk
d. dual-purpose testing

20. Which of the following is correct concerning the required documentation in the working papers of
the performance of the assessment of the risk of material misstatements due to fraud?
a. All risk factors considered should be documented and the response to each documented.
b. Those risk factors identified and the auditor’s response to them should be documented.
c. The major categories of risk factors must be identified, but the particular responses to risk
factors identified need not be documented.
d. No specific documentation is required.

21. If there is fraud involving the collusion of several employees that includes the falsification of
documents, the chance that such a fraud would be uncovered in a normal audit is
a. zero
b. unlikely
c. 50–50
d. very high

22. When the auditor’s regular examination leading to an opinion on the financial statement discloses
specific circumstances that make him suspect that fraud may exist and he concludes that the
results of such fraud, if any, could not be so material as to affect his opinion, he should
a. make a note in his working papers of the possibility of a fraud of immaterial amount so as
to pursue the matter next year.
b. reach an understanding with the client as to whether the auditor or the client, subject to
auditor’s review, is to make the investigation necessary to determine whether fraud has
occurred and, if so, the amount thereof.
c. refer the matter to the appropriate representatives of the clients with the
recommendations that is to be pursued to a conclusion.
d. immediately extend his audit procedures to determine if fraud has occurred and, if so, the
amount thereof.

23. In the regular audit of ABC Company, Mr. X, CPA, discovered a material fraud being perpetrated
by the cashier. What do you expect most of Mr. X to do?
a. Report the incident to the Securities and Exchange Commission.
b. Communicate the existence and details of the fraud to the audit committee of the board
of directors and to at least one managerial level higher than the position occupied by the
fraudster.
c. Advise the shareholders of the client enterprise regarding the fraud.
d. Make an extensive investigation as well as examination in order to account the extent of
the fraud.

24. What differentiates fraud from an error?


a. Materiality
b. Intent
c. Effect on financial statements
d. Frequency of occurrence
AT Quizzer 6 Consideration of Fraud in Audit of FS 4

25. Which of the following acts are considered fraud?


I. Changing of records and documents
II. Misinterpretation of facts.
III. Misappropriation of assets

V. Clerical mistakes

a. III only
b. I and II only
c. I, III and IV only
d. I, II, III, IV and V

26. While performing tax services for a new client, the CPA discovered a material error in a
previously filed tax return prepared by another CPA. In such case, he should:
a. Prepare an affidavit with respect to the error.
b. Recommend compensating for the prior year’s error in the current year’s tax return where
such action will mitigate the client’s cost and inconvenience.
c. Advise the client to file a corrected return regardless of whether or not the error resulted
in an overstatement of tax.
d. Inform the BIR of the error.

27. Which of the following characteristics most likely would heighten an auditor’s concern about risk
of intentional manipulation of financial statements?
a. Turnover of senior accounting personnel is low.
b. Insiders recently purchased additional shares of the entity’s stock.
c. Management places substantial emphasis on meeting earnings projections.
d. The rate of change in the entity’s industry is slow.

28. The regular examination of financial statements is not primarily designed to disclose fraud and
other irregularities although their discovery may result. Normal audit procedures are more likely to
detect a fraud arising from:
a. Forgeries on company checks.
b. Failure to record cash receipts for services rendered.
c. Theft of inventories
d. Collusion on the part of several employees.

29. The auditor, in his plan for an examination in accordance with generally accepted auditing
standards, being influenced by the possibility of material errors will therefore conduct the
examination with an attitude of
a. professional skepticism
b. subjective mistrust
c. objective indifference
d. professional responsiveness
30. An error in which an item is posted to the wrong personal account, or the incorrect calculation of
an amount constituting an original entry is an
a. error of omission
b. error of commission
c. error of principle
d. counter-balancing error

31. A kind of fraud committed by making entry of fictitious payments or failure to enter receipts is
a. misappropriation of goods
b. misappropriation of cash
c. falsification of accounts
d. lapping
32. The practice of withholding receipts from customer(s) of one date and giving the customer(s)
credit at a later date out of cash received from customer is known as:
a. lapping
b. kiting
c. payroll padding
d. window dressing

33. Coverage of shortage in one bank account by means of an unrecorded check drawn on another
bank account is known as:
a. lapping
b. kiting
c. reconciling
d. adjusting

34. It is any illegal misappropriation of bank accounts. Thus, a teller who pockets a portion of his
receipts with intent to defraud the bank for the day is guilty of
a. kiting on bank funds
b. embezzlement
c. abstraction of bank funds
d. lapping of bank funds
AT Quizzer 6 Consideration of Fraud in Audit of FS 5

35. When an independent auditor’s examination of financial statements discloses special


circumstances that make the auditor suspect that material errors and irregularities may exist, the
auditor’s initial course of action should be to
a. Recommend that the client pursue the suspected fraud to a conclusion that is agreeable
to the auditor.
b. Extend normal audit procedures in an attempt to detect the full extent of the suspected
fraud.
c. Reach an understanding with the proper client representative as to whether the auditor or
client is to make the investigation necessary to determine if a fraud has in fact occurred.
d. Decide whether the fraud, if in fact it should exist, might be of such a magnitude as to
affect the auditor’s report on the financial statements.

36. Which of the following most accurately summarizes what is meant by the term “material
misstatement”?
a. Fraud and direct-effect illegal acts
b. Fraud involving senior management and material fraud.
c. Material error, material fraud and certain illegal acts.
d. Material error and material illegal acts.

37. Which of the following illegal acts should an audit be designed to obtain reasonable assurance for
detecting?
a. Securities purchased by relatives of management based on knowledge of inside
information.
b. Accrual and billing of an improper amount of revenue under government contracts.
c. Violations of anti trust laws.
d. Price fixing

38. The most likely explanation why the auditor’s examination cannot reasonably be expected to
bring all illegal acts by the client to the auditor’s attention is that
a. Illegal acts are perpetrated by management override of internal control.
b. Illegal acts by clients often relate to operating aspects rather than accounting aspects.
c. The client’s internal control may be so strong that the auditor performs only minimal
substantive testing.
d. Illegal acts may be perpetrated by the only person in the client’s organization with access
to both assets and the accounting records.

39. What assurance does the auditor provide that errors, irregularities and direct effect illegal
acts that are material to the financial statements will be detected?

Direct Effect Illegal


Errors Irregularities Acts

a.. Limited Negative Limited


b. Limited Limited Reasonable
c. Reasonable Limited Limited
d. Reasonable Reasonable Reasonable

40. Which of the following is not an example of fraud?


a. Entity personnel falsify accounting records.
b. Entity personnel make mistakes in the application of accounting principles.
c. Entity personnel intentionally omit transactions.
d. Entity personnel intentionally misapply accounting principles.

41. Which of the following statements is true?


a. The auditor’s responsibilities to detect errors and fraud in client’s financial statements are
basically the same.
b. The auditor has a greater responsibility to detect errors in client’s financial statements
than to detect fraud in client’s financial statements.
c. The auditor has responsibility to detect errors in client’s financial statements but no
responsibility to detect fraud.
d. The auditor has responsibility to detect fraud in client’s financial statements but no
responsibility to detect errors.

42. Which of the following statements is true?


a. Errors in the financial statements can be ignored because they are intentional.
b. Errors in the financial statements require adjustment of the client’s accounting records.
c. Fraud requires attention of the auditor, but errors do not.
d. Fraud has serious implications only because of their monetary effect on the financial
statements.
AT Quizzer 6 Consideration of Fraud in Audit of FS 6

43. If the auditor has determined that there may be fraud that may have material effect on the
financial statements, all of the following should be done except
a. Consider the implications for other aspects of the audit.
b. Discuss the matter with a level of management where fraud might have occurred.
c. Try to obtain evidence to determine whether the fraud is material and what its effect will
be on the financial statements.
d. If appropriate, suggest that the client consult with legal counsel on matters of law.

44. If necessary, the financial statements should be revised for material fraud or
a. An unqualified or qualified opinion should be issued.
b. A disclaimer of opinion or an adverse opinion should be issued.
c. A qualified or adverse opinion should be issued.
d. An unqualified opinion or a disclaimer of opinion should be issued.

45. Intentional or reckless conduct, whether act or omission, that results in materially misleading
financial statements is called:
a. An error
b. Fraud
c. Fraudulent financial reporting
d. An illegal act

46. A violation of laws or governmental regulations by the audited entity or its management or
employees acting on behalf of the entity is called
a. An error
b. Fraud
c. Fraudulent financial reporting
d. An illegal act

47. The auditor should assess the risk that illegal acts may have a material and direct effect on
financial statements. Based on that assessment, the auditor should
a. Design the audit to provide reasonable assurance of detecting the act.
b. Design the audit to provide absolute assurance of detecting the act because of its nature.
c. Design the audit in the usual way because of the small likelihood of detecting illegal acts.
d. Design the audit in the usual way because illegal acts are outside the scope of the audit.

48. Illegal acts may be


a. Indirect and immaterial
b. Direct and immaterial
c. Direct and material
d. All of the above

49. If the auditor is precluded by the entity from obtaining sufficient appropriate audit evidence to
evaluate whether noncompliance that may be material to the financial statement has, or is likely
to have occurred, the auditor should express
a. unqualified or disclaimer of opinion
b. qualified or adverse opinion
c. disclaimer of opinion or adverse
d. qualified or a disclaimer of opinion

50. Which of the following statements describes why a properly designed and executed audit may not
detect material fraud?
a. Audit procedures that are effective for detecting an unintentional misstatements may be
ineffective for an intentional misstatement that is concealed through collusion.
b. An audit is designed to provide reasonable assurance of detecting material errors, but
there is no similar responsibility concerning material fraud.
c. The factors considered in assessing control risk indicated an increased risk of intentional
misstatements, but only a low risk of unintentional errors in the financial statements.
d. The auditor did not consider factors influencing audit risk for account balances that have
pervasive effects on the financial statements taken as a whole.

51. Which of the following statements best describes an auditor’s responsibility to detect errors
and irregularities?
a. The auditor should study and evaluate the client’s internal control system, and design
the audit to provide reasonable assurance of detecting all errors and irregularities.
b. The auditor should assess the risk that errors and irregularities may cause the financial
statements to contain material misstatements, and determine whether the necessary
internal control procedures have been prescribed and are being followed satisfactorily.
c. The auditor should consider the types of errors and irregularities that could occur, and
determine whether the necessary internal control procedures have been prescribed and
are being followed.
d. The auditor should assess the risk that errors and irregularities may cause the financial
statements to contain material misstatements, and design the audit to provide reasonable
assurance of detecting material errors and irregularities.
AT Quizzer 6 Consideration of Fraud in Audit of FS 7

52. Fraud may be perpetrated with the intent to benefit a company. Which of the following is an
example of such a fraud?
a. Acceptance of bribes or kickbacks by a purchasing agent.
b. Claims submitted for services or goods not actually provided to the company.
c. Sale or assignment of fictitious or misrepresented assets.
d. Diversion by an employee or outsider of a transaction that would normally generate
profits for the company.

53. The risk of management of fraud increases in the presence of


a. Management incentive system based on operating income.
b. Improved internal control.
c. Substantial increases in sales.
d. Frequent changes in suppliers.

54. The risk that an auditor’s procedures will lead to the conclusion that a material error does exist in
an account balance when, in fact, such error does exist, is referred to as:
a. audit risk
b. inherent risk
c. control risk
d. detection risk

55. Jones, CPA, is auditing the financial statements of ABC Retailing, Inc. What assurance does
Jones provide that the audit will detect either direct-effect illegal acts that are material to ABC’s
financial statements or illegal acts that have a material but indirect effect on the financial
statements?

Direct-Effect Indirect-Effect
Illegal Acts Illegal Acts

a. Reasonable None
b. Reasonable Reasonable
c. Limited None
d. Limited Reasonable

56. Which of the following ordinarily is designed to detect possible material peso errors on the
financial statements?
a. Control testing
b. Analytical procedures
c. Computer controls
d. Post-audit working paper review

57. An auditor should recognize that the application of auditing procedures may produce evidential
matter indicating the possibility of errors or irregularities and therefore should
a. Design audit tests to detect unrecorded transactions.
b. Extend the work to audit most recorded transactions and records of an entity.
c. Plan and perform the engagement with an attitude of professional skepticism.
d. Not depend on internal accounting control features that are designed to prevent or detect
errors or irregularities.

58. Which of the following statements describes why a properly designed and executed audit may not
detect a material fraud?
a. Audit procedures that are effective for detecting an unintentional misstatement may be
ineffective for an intentional misstatement that is concealed through collusion.
b. An audit is designed to provide reasonable assurance of detecting material errors, but
there is no similar responsibility concerning material fraud.
c. The factors considered in assessing control risk indicated an increased risk of intentional
misstatements, but only a low risk of unintentional errors in the financial statements.
d. The auditor did not consider factors influencing audit risk for account balances that have
pervasive effects on the financial statements taken as a whole.

59. An auditor concludes that a client’s illegal act, which has a material effect on the financial
statements, has not been properly accounted for or disclosed. Depending on the materiality of the
effect on the financial statements, the auditor should express either a(n)
a. Adverse opinion or a disclaimer of opinion.
b. Qualified opinion or an adverse opinion.
c. Disclaimer of opinion or an unqualified opinion with a separate explanatory paragraph.
d. Unqualified opinion with a separate explanatory paragraph or a qualified opinion.

60. What assurance does the auditor provide that errors, fraud and direct-effect illegal acts that are
material to the financial statements will be detected?
a. Negative
b. Limited
c. Absolute
d. Reasonable
AT Quizzer 6 Consideration of Fraud in Audit of FS 8

61. Which of the following is an incorrect statement?


a. The auditor should assess the risk that errors and fraud may cause the financial
statements to contain material misstatement.
b. The auditor should design the audit to provide reasonable assurance of detecting errors
and fraud that are material to the financial statements.
c. The auditor is not an insurer, and his or her report does not constitute a guarantee.
d. The subsequent discovery that a material misstatement exists in the financial statements
is evidence of inadequate planning, performance or judgment on the part of the auditor.

62. Which of the following statements best describes an auditor’s responsibility to detect errors and
fraud?
a. The auditor should study and evaluate the client’s internal control and design the audit to
provide reasonable assurance of detecting all errors and fraud.
b. The auditor should assess the risk that errors and fraud may cause the financial
statements to contain material misstatements, and determine whether the necessary
internal controls are prescribed and are being followed satisfactorily.
c. The auditor should consider the types of errors and fraud that could occur, and determine
whether the necessary internal controls are prescribed and are being followed.
d. The auditor should assess the risk that errors and fraud may cause the financial
statements to contain material misstatements, and design the audit to provide reasonable
assurance of detecting material errors and fraud.

63. During the annual audit of Ajax Corp., a publicly held company, Jones, CPA , a continuing auditor
determined that illegal political contributions had been made during each of the past seven years,
including the year under audit. Jones notified the board of directors about the illegal contributions,
but they refused to take any action because the amounts were immaterial to the financial
statements.
Jones should reconsider the intended degree of reliance to be placed on the
a. Letter of audit inquiry to the client’s attorney.
b. Prior year’s audit programs.
c. Management representation letter.
d. Preliminary judgment about materiality levels.

64. Which of the following statements reflects an auditor’s responsibility for detecting errors and
fraud?
a. An auditor is responsible for detecting employee errors and simple fraud, but not for
discovering fraud involving employee collusion or management override.
b. An auditor should plan the audit to detect errors and fraud that are caused by departures
from GAAP.
c. An auditor is not responsible for detecting errors and fraud unless the application of
GAAS would result in such detection.
d. An auditor should design the audit to provide reasonable assurance of detecting errors
and fraud that are material to the financial statements.

65. Which of the following characteristics most likely would heighten the auditor’s concern about the
risk of intentional manipulation of financial statements?
a. Turnover of senior accounting personnel is low.
b. Insiders recently purchased additional shares of the entity’s stock.
c. Management places substantial emphasis on meeting earnings projection.
d. The rate of change in the entity’s industry is slow.

66. An auditor concludes that a client has committed an illegal act that has not been properly
accounted for or disclosed. The auditor should withdraw from the engagement if
a. Auditor is precluded from obtaining sufficient competent evidence about the illegal act.
b. Illegal act has an effect on the financial statements that is both material and direct.
c. Auditor cannot reasonably estimate the effect of the illegal act on the financial
statements.
d. Client refuses to accept the auditor’s report as modified for the illegal act.

67. When an auditor becomes aware of a possible illegal act by a client, the auditor should obtain
understanding of the nature of the act to
a. Increase the assessed level of control risk.
b. Recommend remedial actions to the audit committee.
c. Evaluate the effect on the financial statements.
d. Determine the reliability of management’s representations.

68. Which of the following is not an example of an error?


a. Entity personnel make mistakes in gathering or processing accounting data from which a
financial statements are prepared.
b. Entity personnel alter accounting records from which financial statements are prepared.
c. Entity personnel overlook or misinterpret facts, causing accounting estimates to be
incorrect.
d. Entity personnel make mistakes in the application of accounting principles.
AT Quizzer 6 Consideration of Fraud in Audit of FS 9

69. The auditor faces a risk that the examination will not detect material errors that could occur in the
accounting process. In regard to minimizing this risk, the auditor primarily relies on
a. Substantive tests
b. Compliance tests
c. Internal control
d. Statistical analysis

70. An audit should be designed to achieve reasonable assurance of detecting material


a. errors, irregularities and those illegal acts with a direct effect on financial statements
amounts.
b. errors and irregularities.
c. errors
d. errors, irregularities and illegal acts

71. When is the auditor responsible for detecting fraud?


a. When the fraud did not result from collusion.
b. When third parties are likely to rely on the client’s financial statements.
c. When the client’s system of internal control is judged by the auditor to be inadequate.
d. When the application of generally accepted auditing standards would have uncovered the
fraud.

72. An attitude that includes a questioning mind and a critical assessment of audit evidence is
referred to as:
a. Due professional care
b. Professional skepticism
c. Reasonable assurance
d. Supervision

73. Professional skepticism requires the auditor assume that management is


a. Honest, in the absence of fraud risk factors.
b. Dishonest until completion of audit tests.
c. Neither honest nor dishonest.
d. Offering reasonable assurance of honesty.

74. Reportable conditions are matters that come to an auditor’s attention that should
be communicated to an entity’s audit committee because they represent
a. material irregularities or illegal acts perpetrated by high-level management.
b. significant deficiencies in the design or operation of the internal control system.
c. flagrant violations of the entity’s documented conflict-of-interest policies.
d. intentional attempts by client personnel to limit the scope of the auditor’s field work.

75. Which of the following matters is an auditor required to communicate to the entity’s audit
committee?
a. The basis for assessing control risk below the maximum.
b. The process used by management in formulating sensitive accounting estimates.
c. The auditor’s preliminary judgments about materiality levels.
d. The justification for performing substantive procedures at interim dates.

76. Should an auditor communicate the following matters to an audit committee of a public entity?

Significant Audit Adjustments Management’s Consultation


Recorded by the Entity With Other Accountants
About Significant
Accounting Matters

a. Yes Yes
b. Yes No
c. No Yes
d. No No

77. Which of the following statements is correct concerning an auditor’s required communication
with an entity’s audit committee?
a. This communication should include disagreements with management about significant
audit adjustments, whether or not satisfactorily resolved.
b. If matters are communicated orally, it is necessary to repeat the communication
of recurring matters each year.
c. If matters are communicated in writing, the report is required to be distributed to both the
audit committee and management.
d. This communication is required to occur before the auditor’s report on the financial
statements is issued.
AT Quizzer 6 Consideration of Fraud in Audit of FS 10

78. One means of informing the client that the auditor is not responsible for the discovery of fraud is
the
a. engagement letter
b. representation letter
c. responsibility letter
d. client letter

79. Which of the following is correct concerning requirements about auditor communications about
fraud?
a. Fraud that involves senior management should be reported directly to the audit
committee regardless of the amount involved.
b. Fraud with a material effect on the financial statements should be reported directly by the
auditor to the Securities and Exchange Commission.
c. Fraud with a material effect on the financial statements should ordinarily be disclosed by
the auditor through use of an “emphasis of a matter” paragraph added to the audit report.
d. The auditor has no responsibility to disclose fraud outside the entity under any
circumstances.

80. Audits of financial statements are designed to obtain assurance of detecting misstatements due
to

Errors Fraudulent Financial Misappropriation


Reporting of Assets

a. Yes Yes Yes


b. Yes Yes No
c. Yes No Yes
d. No Yes No

*********************

You might also like